Você está na página 1de 65

Document # 6973555

MCAT
Practice Test 7-CBT

i b ut e
D is t r
N ot
Do

1 of 65

Purchased by: Yuri Brito On: 7/18/2012


Document # 6973555

Association of American Medical Colleges

Non-Disclosure Statement

for the

MCAT Practice Test


This practice test is not administered under the same secure conditions as the nationally administered MCAT.
Accordingly, the scores you achieve on this practice test should be considered an estimate of the scores you
might achieve on an actual MCAT administration.

In consideration of being permitted to take this practice test, I affirm that I am bound on my honor to take
the practice test without sharing the content in any form including, printed, electronic, voice, or other means. I
further affirm that I understand that my scores on this practice test are an estimate of the scores I may achieve

t e
on the actual MCAT. I understand that if the AAMC has reason to believe that I have violated this non-

u
disclosure statement, it may, at its discretion, bar me from future practice tests and/or examinations, or take
other appropriate actions.

i b
D is t r
By downloading, printing, or taking this practice test, I acknowledge that I have read this non-
disclosure statement and agree to abide by the terms stated therein.

N ot
Do

2 of 65

Purchased by: Yuri Brito On: 7/18/2012


Document # 6973555

Taking Your Practice Test Offline


The full length practice test may be taken online, printed and taken offline, or a combination of both
methods. However, if you customize your practice test it can only be taken online.

If you started a practice test online, the answer sheet provided at the end of this printout does not include
the answers you entered online. Your online answers will appear on the online answer sheet used to
submit your answers for scoring.

Once you have completed your offline practice test, follow these steps to enter your answers and
submit them for scoring.
Login to the web site.
If this is a new test, click the "Start on Paper" link provided in the "Start a New Test"
table of your home page.
If you want to continue entering answers for an in-progress test, click the "Restart on
Paper" link provided in the "In-Progress Tests" table of your home page.
Click the "Ready to Enter Your Answers? Click Here" link.
Enter your answers in the provided form. Any answers previously entered using your
online practice test or this answer sheet will appear in the form.
Once you have finished entering your answers be sure to save them by clicking "Save",
"Save and Exit", or "Review Online". If you close the answer sheet page without

i b ut e
clicking one of these links, your answers will not be saved.
You may return to the answer sheet to enter or review answers as many times as you like.

r
When you are ready to submit your final answers for scoring, click the "Mark as

t
Complete" link. Once you submit your final answers for scoring, you will not be able to

t D is
review or modify your answers using the entry form.
After your answers have been submitted for scoring, you will automatically return to your
home page. To view your diagnostic score report, click the provided in the "Completed

N
Tests" table.

o
From the diagnostic score report you can review your answers and the solution for each

Do
question.

3 of 65

Purchased by: Yuri Brito On: 7/18/2012


Document # 6973555

Printing Guide

Use this printing guide as a reference to print selected sections of this


practice test.

To print, click the PRINTER icon located along the top of the window and enter one of the
following options in the PRINT RANGE section of the print dialog window:

To Print Enter Print Range Options

Complete Practice Test Click ALL radio button

Click PAGES FROM radio button and


Physical Sciences Section
enter pages 5 to 24

Click PAGES FROM radio button and


Verbal Reasoning Section
enter pages 25 to 39

i b ut e Click PAGES FROM radio button and

t r
Writing Sample Section
enter pages 40 to 42

t D is Click PAGES FROM radio button and

o
Biological Sciences Section
enter pages 43 to 64

Do N Periodic Table
Click PAGES FROM radio button and
enter page 6

Click PAGES FROM radio button and


Answer Sheet
enter page 65

This document has been encoded to link this download to your member account. The AAMC and its
Section for the MCAT hold the copyrights to the content of this Practice Test. Therefore, there can be no
sharing or reproduction of materials from the Practice Test in any form (electronic, voice, or other
means). If there are any questions about the use of the material in the Practice Test, please contact the
MCAT Information Line (202-828-0690).
4 of 65

Purchased by: Yuri Brito On: 7/18/2012


Document # 6973555

Physical Sciences
Time: 70 minutes
Questions: 1-52

Most questions in the Physical Sciences test are organized into groups, each containing a descriptive passage.
After studying the passage, select the one best answer to each question in the group. Some questions are not
based on a descriptive passage and are also independent of each other. If you are not certain of an answer,
eliminate the alternatives that you know to be incorrect and then select an answer from the remaining
alternatives. Indicate your selected answer by marking the corresponding answer on your answer sheet. A
periodic table is provided for your use. You may consult it whenever you wish.

i b ut e
D is t r
N ot
Do
This document has been encoded to link this download to your member account. The AAMC
and its Section for the MCAT hold the copyrights to the content of this Practice Test. Therefore,
there can be no sharing or reproduction of materials from the Practice Test in any form
(electronic, voice, or other means). If there are any questions about the use of the material in the
Practice Test, please contact the MCAT Information Line (202-828-0690).

5 of 65

Purchased by: Yuri Brito On: 7/18/2012


Document # 6973555

1
Periodic Table of the Elements 2
H He
1.0 4.0
3 4 5 6 7 8 9 10
Li Be B C N O F Ne
6.9 9.0 10.8 12.0 14.0 16.0 19.0 20.2
11 12 13 14 15 16 17 18
Na Mg Al Si P S Cl Ar
23.0 24.3 27.0 28.1 31.0 32.1 35.5 39.9
19 20 21 22 23 24 25 26 27 28 29 30 31 32 33 34 35 36
K Ca Sc Ti V Cr Mn Fe Co Ni Cu Zn Ga Ge As Se Br Kr
39.1 40.1 45.0 47.9 50.9 52.0 54.9 55.8 58.9 58.7 63.5 65.4 69.7 72.6 74.9 79.0 79.9 83.8
37 38 39 40 41 42 43 44 45 46 47 48 49 50 51 52 53 54
Rb Sr Y Zr Nb Mo Tc Ru Rh Pd Ag Cd In Sn Sb Te I Xe
85.5 87.6 88.9 91.2 92.9 95.9 (98) 101.1 102.9 106.4 107.9 112.4 114.8 118.7 121.8 127.6 126.9 131.3
55 56 57 72 73 74 75 76 77 78 79 80 81 82 83 84 85 86
Cs Ba La* Hf Ta W Re Os Ir Pt Au Hg Tl Pb Bi Po At Rn
132.9 137.3 138.9 178.5 180.9 183.9 186.2 190.2 192.2 195.1 197.0 200.6 204.4 207.2 209.0 (209) (210) (222)
87 88 89 104 105 106 107 108 109 110 111 112 114 116
Fr Ra Ac† Rf Db Sg Bh Hs Mt Ds Uuu Uub Uuq Uuh
(223) (226) (227) (261) (262) (266) (264) (277) (268) (281) (272) (285) (289) (289)

58
* Ce
140.1
59
Pr
140.9
60
Nd
144.2
61
Pm

i
(145)

b ut
Sm

e
62

150.4
63
Eu
152.0
64
Gd
157.3
65
Tb
158.9
66
Dy
162.5
67
Ho
164.9
68
Er
167.3
69
Tm
168.9
70
Yb
173.0
71
Lu
175.0

t r
90 91 92 93 94 95 96 97 98 99 100 101 102 103
† Th

s
Pa U Np Pu Am Cm Bk Cf Es Fm Md No Lr

i
232.0 (231) 238.0 (237) (244) (243) (247) (247) (251) (252) (257) (258) (259) (260)

N ot D
Do

6 of 65

Purchased by: Yuri Brito On: 7/18/2012


Document # 6973555

Passage I
Like ammonia, hydrazine is a base in aqueous
Thousands of tons of hydrazine (N2H4) are produced solution. Figure 1 shows the equilibria reactions of
each year for commercial uses, including the ammonia and hydrazine in aqueous solution.
production of agricultural chemicals. At room
temperature, hydrazine is a volatile liquid that exists NH3(aq) + H2O(ℓ) NH4+(aq) + OH-(aq)
in hydrogen-bonded networks similar to those found Keq = 1.8 × 10-5
in liquid water. Hydrazine may be prepared by the
Raschig process, the reaction of ammonia with
N2H4(aq) + H2O(ℓ) N2H5+ + OH-(aq)
sodium hypochlorite, as shown in Equation 1.
Keq = 8.5 × 10-7
2NH3(g) + NaOCl(aq) → N2H4(aq) + NaCl(aq) +
H2O(ℓ) N2H5+ + H2O N2H62+ + OH-(aq)
Keq = 8.9 × 10-16
Equation 1
Figure 1 Equilibria (Keq = equilibrium constant)
Hydrazine usually is shipped as the hydrate (N2H4 ·
H2O) because it is easier to handle and can be easily 1. Which of the following Lewis structures best
dehydrated to form the anhydrous compound. represents hydrazine?
A)

e
Hydrazine and its chemical derivatives are good
rocket propellants. For example, hydrazine reacts
with dinitrogen tetroxide (N2O4) to produce gaseous
nitrogen and water. Equation 2 shows the reaction and

ri b ut
the enthalpy change.

2 N2H4(ℓ) + N2O4(ℓ) → 3 N2(g) + 4 H2O(g)

D is t B)

N
∆H° = -1040 kJ mol-1

Equation 2
ot
Do
C)
Some thermochemical data for hydrazine and
dinitrogen tetroxide are given in Table 1.

Table 1 Properties of Hydrazine and Dinitrogen


D)
Tetroxide at 298 K
Property N2H4(ℓ) N2O4(g)
-1
∆Hf° (kJ mol ) 50.6 9.2
∆Gf° (kJ mol-1) 149.2 97.9
S° (J K-1 mol-1) 121.2 304.3 2. How many grams of ammonia are required to
make one mole of hydrazine by the Raschig
process?
A) 8.5 g
B) 17.0 g
C) 32.0 g
D) 34.0 g

Sharing or reproducing this material in any form is a violation of the AAMC copyright Page 7 of 65

Purchased by: Yuri Brito On: 7/18/2012


Document # 6973555

5. The formation of hydrazine from its elements is


3. NOT a spontaneous process at 25oC and 1 atm
because:
What is the enthalpy change (ΔHo) for the reaction
A) ΔSo for the reaction is > 0.
shown above?
B) ΔHo for the reaction is < 0.
A) 50.6 kJ mol-1
C) ΔGo for the reaction is > 0.
B) 149.2 kJ mol-1
D) So for hydrazine is > 0.
C) (149.2 + 298 x 121.2) kJ mol-1
D) (149.2 - 50.6) kJ mol-1 6. The entropy change (ΔSo) for the reaction shown
in Equation 2 is:
4. As a result of being a weaker base than ammonia,
A) < 0 because the moles of gaseous products > the
hydrazine:
moles of gaseous reactants.
A) has a smaller acidity constant (Ka) than does
B) < 0 because water is a product of the reaction.
ammonia.
C) > 0 because the moles of gaseous products > the
B) has a smaller basicity constant (Kb) than does
moles of gaseous reactants.
ammonia.
D) > 0 because water is a product of the reaction.
C) can be protonated twice to form N2H62+.
D) forms hydrogen bonds in aqueous solution.

i b ut e
D is t r
N ot
Do

Sharing or reproducing this material in any form is a violation of the AAMC copyright Page 8 of 65

Purchased by: Yuri Brito On: 7/18/2012


Document # 6973555

Passage II

A gas of electrically charged and neutral particles is


called a plasma. Plasma physics is a broad term
applicable to such diverse areas as space physics, gas
lasers, gaseous electronics, and controlled
thermonuclear fusion.

A plasma has the ability to oscillate and propagate


waves. These waves can be excited by applying an
oscillating electric field to the plasma. The simplest
oscillation is a high-frequency oscillation of the
plasma electrons. Consider a plasma that is
electrically neutral, consisting of positive ions
immersed in a “sea” of electrons. If the electron sea
is slightly displaced from the ionic background,
electric fields act to restore the electrons to their
original equilibrium positions. The electron sea
subsequently moves toward the equilibrium position,
overshoots, and oscillates back and forth. These

e
oscillations are so rapid that the positive ions seem to

t
Figure 1 Positive ions surrounded by a sea of

u
be fixed in the background (see Figure 1). The
mobile electrons (gray denotes the electron sea).

b
frequency f at which these oscillations occur for a

ri
given number density, n (electrons per cubic meter), A and C denote the oscillation extremes.
is
2
f = [kne /(πm)] 1/2

D
≈ 9.0n
is t
1/2
7. Why can the positive ions be considered to be
fixed during the electrons’ oscillations?

N ot
where e and m are the elementary charge and electron
mass, 1.6 x 10-19 C and 9 x 10-31 kg, respectively. The
A) The ions are bound together with strong nuclear
forces.

Do
constant k = 9 x 109 Nm2/C2 occurs in Coulomb’s B) An ion is much more massive than an electron.
law. The approximation on the right side of the C) The ions experience no force when the electron
equation gives the frequency in Hz, when n is sea is displaced.
expressed in m-3.
D) Coulomb’s law prohibits the motion of the ions.

8. In Figure 1, the maximum electrical potential


energy occurs at:
A) A only.
B) B only.
C) C only.
D) A and C only.

Sharing or reproducing this material in any form is a violation of the AAMC copyright Page 9 of 65

Purchased by: Yuri Brito On: 7/18/2012


Document # 6973555

9. The density of a typical laboratory plasma is 1018 11. As the Figure 1 electrons oscillate through
m-3. This value leads to plasma oscillations at: equilibrium point B, they move on to C because
of:
A) 9 x 1018 Hz.
A) the momentum gathered as they moved from point
B) 9 x 1012 Hz.
A.
C) 9 x 109 Hz.
B) Coulomb forces pulling on the electron sea.
D) 9 x 106 Hz.
C) magnetic forces of attraction between the positive
ions and the electron sea.
10. A plasma wave moving through a plasma has a
frequency of 109 Hz and a speed of 3.0 x 107 D) the large potential energy they have at point B.
m/s. What is the wavelength of this wave?
A) 3.0 cm 12. What best describes changes that occur as the
electron sea moves from position A to position B
B) 3.0 m in Figure 1?
C) 3.3 cm A) Kinetic energy is transformed into potential
D) 3.3 m energy.
B) Potential energy is transformed into kinetic
energy.

i b ut e C) Power is dissipated as heat.


D) Turbulence brings the electron sea to rest.

D is t r
N ot
Do

Sharing or reproducing this material in any form is a violation of the AAMC copyright Page 10 of 65

Purchased by: Yuri Brito On: 7/18/2012


Document # 6973555

Passage III 14. What is the electron configuration for a ground-


state silicon atom?
Silicon, the second most abundant element in the
earth’s crust, is found combined with oxygen in a A) [Ne] 3s ↑↓ 3p ↑↑ __
variety of silicate minerals. The most common is B) [Ne] 3s ↑↓ 3p ↑ ↓ _
silica (SiO2), which is a network solid.
C) [Ne] 3s ↑↓ 3p ↑↓ __ _
Silicon cannot be purified by electrolytic techniques. D) [Ne] 3s ↑↓ 3p↓ ↓ ↓
When elemental potassium became available in the
nineteenth century, it was used in a silicon
purification procedure. Today, silicon is produced 15. According to valence shell electron pair repulsion
commercially by the reaction of silica with carbon or (VSEPR) theory, what is the geometry around
calcium carbide in an electric furnace at 2000°C silicon in SiCl3H?
(Equation 1). The product is about 98% pure, with A) Linear
impurities of iron, oxygen, aluminum, and other
elements. Further purification is achieved by B) Tetrahedral
halogenating the silicon, purifying the resulting gas C) Trigonal bipyramidal
by fractional distillation, and then reducing the
halogenated silicon compound (Equations 2-3). D) Octahedral

e
SiO2(s) + 2 C(s) → Si(ℓ) + 2 CO(g) 16. Which of the following elements could best

Equation 1

ri b ut substitute for potassium in the purification of


silicon?

t
A) H2

s
Si(s) + 3 HCl(g) → SiCl3H(g) + H2(g)

Equation 2

SiCl3H(g) + H2(g) → Si(s) + 3 HCl(g)

ot D i B) Na
C) Mg
D) Ca

Do N Equation 3

Pure silicon is a hard, brittle, nonreactive substance


with a metallic luster.
17. SiCl3H has a normal boiling point of 33oC. What
are the predominant forces between SiCl3H
molecules?
A) Ionic forces
13. The purification of elemental silicon was difficult B) Covalent bonds
to achieve because it: C) Hydrogen bonds
A) is a rare element. D) van der Waals forces
B) is too reactive to isolate easily.
C) exists in minerals that do not decompose easily.
D) does not crystallize.

Sharing or reproducing this material in any form is a violation of the AAMC copyright Page 11 of 65

Purchased by: Yuri Brito On: 7/18/2012


Document # 6973555

18. SiCl3H is purified by fractional distillation. Why


does this procedure effect a purification?
A) SiCl3H is not water soluble.
B) SiCl3H is decomposed by water.
C) SiCl3H has a lower boiling point than the solid
impurities.
D) SiCl3H has a lower melting point than the
impurities.

i b ut e
D is t r
N ot
Do

Sharing or reproducing this material in any form is a violation of the AAMC copyright Page 12 of 65

Purchased by: Yuri Brito On: 7/18/2012


Document # 6973555

These questions are not based on a descriptive 21. If there is no air resistance, how far will a 2-kg
passage and are independent of each other. object fall from rest in 10 sec?
(Note: Use g = 10 m/s2.)
19. Consider the following electrode potentials. A) 100 m
Cu2+ + 2 e- → Cu(s) Eo= +0.34 V B) 250 m
C) 300 m
2 H2O → O2 + 4 H+ + 4 e- Eo = -1.23 V
D) 500 m
What is Eocell
for the reaction shown in the
following equation? 22. When a light wave and a sound wave pass from
air to glass, what changes occur in their speeds?
2 Cu2+ + 2 H2O → 2 Cu(s) + O2 + 4 H+
A) Both speed up.
A) -0.89 V
B) Both slow down.
B) +0.55 V
C) Light speeds up; sound slows down.
C) +1.57 V
D) Light slows down; sound speeds up.
D) +1.91 V

e
20. A gas that occupies 10 L at 1 atm and 25oC will
occupy what volume at 500 atm and 25oC?
A) Exactly 0.020 L

ri b ut
B) Somewhat more than 0.02 L because of the space
occupied by the individual gas molecules

D is t
t
C) Somewhat more than 0.02 L because of the

N
D) Somewhat more than 0.02 L because of the
o
repulsions between the individual gas molecules

Do
increased number of collisions with the sides of
the container

Sharing or reproducing this material in any form is a violation of the AAMC copyright Page 13 of 65

Purchased by: Yuri Brito On: 7/18/2012


Document # 6973555

Passage IV

When aqueous solutions of bromine and acetone are mixed, the reaction shown by Equation 1 occurs.

Equation 1

When the pH of the solution is between 4 and 7, the reaction occurs very slowly. However, at pH values less
than 3, the reaction occurs rapidly.

If the bromination of acetone (molar mass = 58.0 g mol–1 and density = 0.791 g mL–1) follows simple kinetics,
the rate law can be expressed by Equation 2.

Rate = –Δ[Br2]/Δt = k[acetone]a[Br2]b[H+]c

Equation 2

Bromine is a red–brown liquid that absorbs light very strongly at a wavelength of 395 nm, and it is the only

t e
compound that absorbs visible light during this reaction. Thus, a researcher can use a spectrophotometer to

u
follow the decrease in the concentration of bromine. The amount of 395-nm light absorbed by bromine is

b
directly proportional to the concentration of bromine. Equation 3 is Beer’s law, which shows the relationship

t ri
between the absorbance A and the concentration c of the absorbing species when light passes through a cuvette

s
of path length l. The molar absorbtivity ε is a constant for a given wavelength, and the path length is normally

i
D
1.00 cm.

N ot А = ε cl

Do
Equation 3

Table 1 gives rate data for this reaction. In the experimentally determined rate law, the reaction is zero order
with respect to bromine.

Table 1 Rate Data at 25 C and 395 nm

Experiment [acetone] [H+] [Br2] – Δ[Br2]/Δt rate constant


Number M M M M s–1 k 10–5
1 1.60 0.403 4.14 10 3
28.0 10–6 4.35
2 0.80 0.101 3.96 10 3
2.85 10–6 3.52
3 0.40 0.202 3.69 10 3
2.94 10–6 3.65
4 0.80 0.403 4.26 10 3
12.9 10–6 4.00
5 1.60 0.202 4.38 10 3
12.7 10–6 3.93
6 0.80 0.202 4.28 10 3
5.99 10–6 3.70
–5 –1 –1
kave = 3.86 0.2 10 M s

Sharing or reproducing this material in any form is a violation of the AAMC copyright Page 14 of 65

Purchased by: Yuri Brito On: 7/18/2012


Document # 6973555

25. If the reaction is first order with respect to both


23. The molar absorptivity of bromine at 395 nm is acetone and hydronium ion, which of the
198 M–1 cm–1. What is the absorbance at 395 nm following equations gives the rate law?
in Experiment 1? A) Rate = k[acetone][H+]
A) 0.00825 B) Rate = k[acetone][Br2]2[H+]
B) 0.820 C) Rate = k[acetone][Br2][H+]
C) 1.22 D) Rate = k[acetone]2[H+]
D) 20.9
26. What is the molarity of pure acetone?
24. What is the value of c in Equation 2 as A) 1.36 M
determined from the data in Table 1?
B) 13.6 M
A) 1
C) 45.9 M
B) 2
D) 73.4 M
C) 3
D) 4 27. Though 395-nm light is in the visible region of
the electromagnetic spectrum, it is very near:

i b ut e A) the radio wave region.


B) the microwave region.

D is t r C) the infrared region.


D) the ultraviolet region.

N ot
Do

Sharing or reproducing this material in any form is a violation of the AAMC copyright Page 15 of 65

Purchased by: Yuri Brito On: 7/18/2012


Document # 6973555

Passage V 29. If a cellular phone is powered by a 12-volt


battery and is transmitting at its maximum power,
Cellular phones are commonly used by people who what current is being used?
are traveling away from home or on business. The
development of these phones combines many recent A) 3 W
innovations in technology. Simply described, cellular B) 3 A
phones behave like two-way radios with the incoming
C) 0.25 A
voice data transmitted at one carrier frequency and
the outgoing voice data transmitted at another D) 0.05 A
frequency. This capability to send and receive voice
data on these two separate frequency channels allows 30. If two people were talking on their cellular
the person to hear and speak on the phone at the same phones within the same "cell" in a city, why
time. The information in these channels is would their transmissions NOT interfere?
transmitted on radio-frequency electromagnetic
carrier waves, which travel well through the air. A) The power transmitted from each phone is not
high enough to interfere.
Cell-phone channels operate at frequencies ranging B) The signals are transmitted at the speed of light,
between 824 MHz and 894 MHz. Each channel and do not have time to interfere.
requires a finite amount of frequency space, called the
C) The physical distance between any two base
bandwidth of the channel, and is set at 30 kHz. Most
stations limits interference.

e
cellular phones can transmit their signal with between
0.6 watts and 3 watts of power. The cell phone scans
all of its channels when it is on to find the channel
with the highest signal intensity. The phone

ri b ut D) The frequencies used by each phone are chosen to


be different.

communicates with a base station, which typically


covers an area of 10 square miles, called a “cell.” A
cellular city has many “cells” within it, which have

D is t 31. The intensity of a cellular phone transmission


received at the switching station is proportional

t
to the power used by the phone and inversely

o
phone base stations to transmit and receive cell-phone proportional to the square of the distance between

N
data. Because the size of a cell is relatively small, it phone and station. Which combination of power
allows efficient communication with relatively low

Do
and distance will provide the highest signal to be
power phones. When a phone moves from cell to picked up by the switching station?
cell, its calls are handled by a central switching office.
A) 0.6 watts, 2 miles
28. What is the total frequency range available for B) 0.6 watts, 3 miles
cellular phone communications? C) 3 watts, 5 miles
A) 30 kHz D) 3 watts, 4 miles
B) 894 MHz
C) 70 MHz
D) 894.03 MHz

Sharing or reproducing this material in any form is a violation of the AAMC copyright Page 16 of 65

Purchased by: Yuri Brito On: 7/18/2012


Document # 6973555

32. The 846 MHz carrier wave is an electromagnetic


signal, whereas sound waves are typically at
much lower frequency and are carried through
the air as pressure waves. Which statement
describes the two waves accurately?
A) Electromagnetic waves are transverse; pressure
waves are longitudinal.
B) Electromagnetic waves are longitudinal; pressure
waves are transverse.
C) Waves are always longitudinal and transverse.
D) Electromagnetic wavelengths are longer than
acoustic sound wavelengths.

i b ut e
D is t r
N ot
Do

Sharing or reproducing this material in any form is a violation of the AAMC copyright Page 17 of 65

Purchased by: Yuri Brito On: 7/18/2012


Document # 6973555

Passage VI
The exchange of oxygen-18 between H218O and SO42-
Many reactions of oxyanions (negative ions that is also more rapid in acid than in neutral solutions of
contain oxygen) involve the transfer of oxygen atoms SO42-. A proposed reaction mechanism for the
from one ion or molecule to another. exchange is shown below.

Reaction 1 shows an oxygen atom transfer that is Sequence I 2 H+ + SO42- H2SO4 (fast)
typical of an oxyanion reaction. Sequence II H2SO4 (slow)
SO3 + H2O
NO2- + OCl- NO3- + Cl- Keq = 1068 Sequence III SO3 + H218O 2 H+ + (fast)
SO318O2-
Reaction 1
33. If the rate of formation of Cl- in Reaction 3 were
Despite the favorable equilibrium constant, this 1.0 x 10-2M/sec at a pH of 1, what would it be at
reaction is extremely slow. The reaction rate can be a pH of 2? (Note: Assume that other conditions
increased by adding acid to the reaction solution. are identical.)
When added, acid reacts with OCl-, forming HOCl.
A) 1 x 10-1M/s
HOCl allows the oxygen transfer to take place more
quickly because the hydrogen atom reduces the B) 1 x 10-2M/s
charge on the oxygen atom, facilitating the breaking C) 2 x 10-2M/s

e
of the O-Cl bond. The rate of this reaction, Reaction

t
2, is first order in both NO2- and HOCl. D) 1 x 10-4M/s

NO2- + HOCl NO3- + Cl- + H+

s t ri b u
Keq = 1043 34. Compared to the rate of Reaction 1, the rate of

i
Reaction 2:

D
Reaction 2 A) is 1025 times less.
Other oxyanion reactions also take place more

N
quickly in acidic solutions. For example, no

ot B) is 1025 times greater.


C) is 1.58 times greater.

Do
observable reaction occurs between ClO3- and Br- in
basic solution, but when an acidic solution is used, D) cannot be evaluated without additional
Reaction 3 occurs rapidly. information.

ClO3- + 6 Br- + 6 H+ → Cl- + 3 Br2 + 3 H2O

Reaction 3

The rate law for Reaction 3 is k[ClO3-][Br-][H+]2, and


the initial sequences of the reaction mechanism are
shown below.

Sequence I 2 H+ + ClO3- H2OClO2+ (fast)


Sequence II Br- + H2OClO2+ BrClO2 (slow)
+ H2O
Sequence III Br- + BrClO2 Br2 + ClO2- (fast)

Sharing or reproducing this material in any form is a violation of the AAMC copyright Page 18 of 65

Purchased by: Yuri Brito On: 7/18/2012


Document # 6973555

35. In addition to the explanation in the passage, the 37. Which of the following figures represents a likely
rate of Reaction 2 is different from the rate of transition state for Reaction 2?
Reaction 1 because the formation of HOCl by the A)
protonation of the oxygen:
A) reduces the electronic repulsion forces between
the reactants.
B) increases the electronic repulsion forces between
the reactants. B)
C) increases the electronic repulsion forces between
the nitrogen atom and the oxygen atom that is
being transferred.
D) reduces the electronic attraction forces between
the chlorine atom and the oxygen atom that is C)
being transferred.

36. Which of the following methods would produce


SO318O2- at the fastest rate?
D)
A) Bubbling SO3(g) through H218O
B) Bubbling S18O3(g) through H218O
C) Bubbling S18O3(g) through H2O

i b ut e
D) Reacting SO218O(l) with H218O

D is t r
N ot
Do

Sharing or reproducing this material in any form is a violation of the AAMC copyright Page 19 of 65

Purchased by: Yuri Brito On: 7/18/2012


Document # 6973555

38. Which of the following graphs best shows the


energy diagram for the Reaction 3 mechanism in
the passage?
A)

B)

C)

i b ut e
D is t r
N ot
DoD)

Sharing or reproducing this material in any form is a violation of the AAMC copyright Page 20 of 65

Purchased by: Yuri Brito On: 7/18/2012


Document # 6973555

These questions are not based on a descriptive


passage and are independent of each other. 41. A ray of light in air is incident upon a glass plate
at an angle of 45o. The angle of refraction of the
39. Phosphorus appears directly below nitrogen in ray in the glass is 30o. What is the index of
the periodic table. The boiling point of ammonia, refraction of the glass?
NH3, is higher than the boiling point of
phosphine, PH3, under standard conditions. (Data: sin 30o= 0.500, sin 45o= 0.707,
Which of the following statements best explains sin 60o= 0.866, tan 30o= 0.577,
the difference in the boiling points of these two tan 45o= 1.000)
compounds? A) 1.22
A) Ammonia is a weaker base than phosphine. B) 1.41
B) The N-H bond is weaker than the P-H bond. C) 1.57
C) High molecular weight compounds generally have D) 1.65
lower boiling points.
D) Ammonia forms stronger intermolecular hydrogen
bonds than phosphine.

40. A student measures the mass and volume of four

t e
objects.

Object Mass(g) Volume(cm3)


A 1.5 0.50

s t ri b u
i
B 3.0 0.75

D
C 4.5 1.00

t
D 6.0 1.50

N Which object has the highest density?


o
Do
A) A
B) B
C) C
D) D

Sharing or reproducing this material in any form is a violation of the AAMC copyright Page 21 of 65

Purchased by: Yuri Brito On: 7/18/2012


Document # 6973555

Passage VII 42. Before the Landers quake, coincidence was


argued as a sufficient explanation for what is now
Worldwide, about 20 damaging earthquakes occur believed to be triggered-quake events. For these
daily. A major quake in the Mojave Desert (near earlier events, which of the following does NOT
Landers) in 1992 demonstrated that large quakes support the coincidence hypothesis?
sometimes trigger distant smaller ones. Of the many
seismographs installed throughout the West in the A) The timing of subsequent quakes provided
1980s, 14 recorded local quakes after the Landers ambiguous evidence.
event, making coincidence an unlikely explanation. B) Generally, the distances to the subsequent quakes
was excessive.
The Landers quake produced measurable lasting
C) Too few of the subsequent quakes were recorded
deformations over a length L = 74 km. L is called the
to establish a clear connection.
source length of the initiating quake. Allied quakes,
aftershocks, occur within a distance of 2L from the D) Generally, the subsequent quakes were scattered in
primary event. However, triggered quakes were as all directions.
far away as 17L.
43. What is the ratio of lasting deformations
The mechanism of the triggering is a puzzle. It is produced by a quake at 16 L from its center to
useful to categorize seismic waves into two kinds: those produced at 4 L from its center?
deeply propagating body waves, which dissipate
A) 1/16

e
rapidly with distance, and surface waves, which
dissipate at a lesser rate. Lasting deformations from a
quake are produced by the body waves, with
deformation size falling off as (L/d)3, where d is

ri b ut B) 1/32
C) 1/64
distance from the quake center. At d = 4L these
deformations are generally reduced to less than the

D is t D) 1/128

t
daily periodic distortions due to tidal forces. On the 44. Because earthquakes were triggered by what

o
other hand, surface waves are associated with elastic ultimately must have been comparatively minor

N
oscillations in the crust of about 10-s periodicity and energy transfers from the distant Landers quake,
cause little lasting deformation. Some of the time

Do
the triggered quakes probably occurred where:
delays between the Landers quake and those it
triggered were too great to be ascribed to seismic A) significant local stress forces in the earth’s crust
wave-transit times. were already in precarious equilibrium.
B) the earth’s crust was subjected to reinforcing
One explanation of the aftershock trigger mechanism resonant effects that cumulatively built up local
involves underground fluids, water or molten rock. stresses.
Fluid seals between isolated volumes of rock at
different pressures may leak, thus increasing C) the earth’s crust locally sustained standing wave
transverse frictional forces. Also, fluid may flow into nodes for a short but significant period.
rock fractures, thus lubricating them. D) destructive interference effects in the incoming
seismic waves were approximately maximum for
an extended period.

Sharing or reproducing this material in any form is a violation of the AAMC copyright Page 22 of 65

Purchased by: Yuri Brito On: 7/18/2012


Document # 6973555

45. The wavelength of surface waves is about 20 46. The primary rupture in the Landers quake moved
km. The propagation speed of these waves is from south to north as it progressed to its full 74-
estimated as: km length. How would the Doppler effect
influence this?
A) hundreds of m/s.
A) By decreasing the wavelength of seismic waves
B) thousands of m/s.
propagating eastward and westward
C) tens of thousands of m/s.
B) By increasing the wavelength of seismic waves
D) hundreds of thousands of m/s. propagating eastward and westward
C) By decreasing the wavelength of seismic waves
moving northward and increasing the wavelength
of waves moving southward
D) By increasing the wavelength of seismic waves
moving northward and decreasing the wavelength
of waves moving southward

i b ut e
D is t r
N ot
Do

Sharing or reproducing this material in any form is a violation of the AAMC copyright Page 23 of 65

Purchased by: Yuri Brito On: 7/18/2012


Document # 6973555

These questions are not based on a descriptive


passage and are independent of each other. 50. A ray of light in air strikes the flat surface of a
liquid, resulting in a reflected ray and a refracted
47. What is the difference in pressure between two ray. If the angle of reflection is known, what
points that are separated by a vertical distance of additional information is needed in order to
0.25 m in a tank of water? (Note: The density of determine the relative refractive index of the
water is 1,000 kg/m3, and g = 10 m/s2.) liquid compared to air?
A) 250 N/m2 A) Angle of incidence
2
B) 400 N/m B) Angle of refraction
2
C) 2,500 N/m C) Refractive index of air
2
D) 4,000 N/m D) Wavelength of the light

48. The following reaction occurs spontaneously. 51. Approximately how many moles of Al3+ are
reduced when 0.1 faraday of charge passes
Cd(s) + 2 H+(aq) → Cd2+(aq) + H2(g) through a cell during the production of Al? (Note:
Assume there is excess Al3+ available and that
Which of the following has the highest electron Al3+ is reduced to Al metal only.)
affinity? A) 0.033 mol
A) Cd(s)
+
B) H (aq)

i b ut e B) 0.050 mol
C) 0.067 mol

t r
2+
C) Cd (aq)

s
D) 0.10 mol
D) H2(g)

49. The energy, E, of a hydrogen atom with its

ot D i 52. When a weak acid (HA) is titrated with sodium


hydroxide in the presence of an indicator (HIn),
the pH at which a color change is observed

N
electron in the nth shell of a hydrogen atom is depends on the:

Do
given by E = -C/n2 where n = 1,2,3, . . . and C is a A) final concentration of HA.
positive constant. If an electron goes from the n
= 2 shell to the n = 3 shell: B) final concentration of HIn.

A) a photon is emitted. C) initial concentration of HA.

B) an electron is emitted. D) pKa of HIn.

C) an electron is absorbed.
D) the energy of the atom is increased.

Sharing or reproducing this material in any form is a violation of the AAMC copyright Page 24 of 65

Purchased by: Yuri Brito On: 7/18/2012


Document # 6973555

Verbal Reasoning
Time: 60 minutes
Questions: 53-92

There are seven passages in the complete Verbal Reasoning test. Each passage is followed by several
questions. After reading a passage, select the one best answer to each question. If you are not certain of an
answer, eliminate the alternatives that you know to be incorrect and then select an answer from the remaining
alternatives. Indicate your selected answer by marking the corresponding answer on your answer sheet.

i b ut e
D is t r
N ot
Do
This document has been encoded to link this download to your member account. The AAMC and its Section
for the MCAT hold the copyrights to the content of this Practice Test. Therefore, there can be no sharing or
reproduction of materials from the Practice Test in any form (electronic, voice, or other means). If there are
any questions about the use of the material in the Practice Test, please contact the MCAT Information Line
(202-828-0690).

Purchased by: Yuri Brito On: 7/18/2012


Document # 6973555

Passage I whether it is edible, combustible, domesticable, or


whatever.
Students of evolution have shown that species death,
or extinction, is going on all the time and that it is an I am concerned with an image of our species as a
essential feature of life history. What alarms so many vast, featureless mob of Yahoos mindlessly trampling
life historians is not that extinctions are occurring but this planet’s most ancient and delicate harmonies.
that they appear to be occurring at a greater rate than This image is not a completely inaccurate description
they have at all but a few times in the past, raising the of present conditions in some parts of the world, but it
specter of the sort of wholesale die-offs that ended the portrays the human presence as a sort of monolithic
reign of the dinosaurs. A new word has been coined disaster, when in fact Homo sapiens is the crown of
to define the value most threatened by these creation. Change is one of the most reliable constants
overheated rates: biodiversity. of this story. To say that the changes we have
brought, and will continue to bring, are somehow
Since extinction is a particularly final and alien to the world and are within a half inch of
comprehensive form of death, species preservation making its “natural” continuance impossible displays
and its corollary, habitat protection, are now seen as some contempt for the forces at work.
the most important means available to stem the
erosion of biodiversity, but I wonder if these ideas, Today, many believe that these changes are often for
which emphasize diversity at the species level, give the worse. We look back with longing to a time at
an adequate picture of recent biological history. If we which the human presence barely dimpled the

e
believe that all life shares a certain quality of landscape. I’m not persuaded by this picture. I can
sensitivity, or self-awareness, then Homo sapiens was
an astonishing and wholly unpredictable leap forward
in this respect, because human beings manifested an

ri b ut easily imagine arguments that would have required


the interior of North America to remain empty of
cities and yet I don’t think this continent is a poorer
idea of personhood never before achieved.

D is t place now than it was twenty thousand years ago.


The more convinced we are that our species is a

t
Consciousness. Mind. Insight. Here are qualities plague, the more we are obliged to yearn for disasters.

o
that, if not exclusively human, seem appallingly

N
rudimentary elsewhere. Plainly, our planet contained Material used in this test passage has been adapted from the
vast opportunities for creatures willing to shape it following source:

Do
T. Palmer, The case for human beings. ©1992 by T. Palmer.
consciously toward their ends. The way was clear;
we know of no other species that has divined what
we’ve been up to or has a mind to object. What 53. Which of the following statements best
seems simple to us is far beyond them; it’s almost as summarizes the central thesis of the passage?
if we move so fast that we are invisible, and they are A) Biodiversity is essential for human survival.
still trying to pretend that the world is the same as it
was before we arrived. B) Cultural adaptation and genetic adaptation are
separate forms of evolution.
This speed on the uptake appears to be the chief C) Changes brought on by human judgment are
advantage that cultural adaptation has over genetic. natural.
When human beings encounter new circumstances,
D) Extinction is inevitable.
adaptation rarely depends on which individuals are
genetically best suited to adjust, passing on their
abilities more successfully than others and producing
subsequent generations better adapted to the new
order. No, human beings tend to cut the loop short by
noticing the new, puzzling over it, telling their
friends, and attempting to find out immediately

Sharing or reproducing this material in any form is a violation of the AAMC copyright Page 26 of 65

Purchased by: Yuri Brito On: 7/18/2012


Document # 6973555

54. Assume that a new species is discovered and 57. According to the author, how do many life
found to have a capacity for reasoned judgment historians support their contention that we risk
greater than that of human beings. The author destroying species?
would be most likely to argue that: A) They assert that extinctions are occurring at a
A) the new species will be highly adaptable. rapidly accelerated rate.
B) adaptation is taking place at a faster rate than B) They maintain that extinctions are brought about
scientists had previously thought. only by human intervention.
C) biological classifications should be altered. C) They cite examples of extinct species.
D) decreasing biodiversity will destroy the D) They point to similarities between humans and
environment. dinosaurs.

55. The author apparently believes that the increasing 58. Which of the following processes would the
rate of extinction: author be most likely to characterize as “cultural
adaptation”?
A) will decrease naturally.
A) Getting to know people from different cultures
B) should be halted.
B) Moving around to many different regions of the
C) has nothing to do with human activity.
world

e
D) is not a reason for panic.

t
C) Reasoning and problem solving to change a

u
situation

i b
56. Suppose that a conference is held on issues

r
D) Moving to a culture that one finds suitable

t
relevant to the transformation of Mars into a

is
habitable planet. Which of the following topics

D
is the author most likely to present?
A) The importance of preserving native habitats on

N
Mars

ot
Do
B) The importance of farming species likely to
survive on Mars
C) The role of human inventiveness in ensuring
survival on Mars
D) Reasons that biodiversity is irrelevant on Mars

Sharing or reproducing this material in any form is a violation of the AAMC copyright Page 27 of 65

Purchased by: Yuri Brito On: 7/18/2012


Document # 6973555

Passage II faculties imagination, which enabled the artist to


transform the objects of the visible world and thus to
There is no doubt that what we call the modern create an ideal space occupied by ideal forms; or
movement in art begins with the single-minded intellect, which enabled the artist to construct a
determination of a French painter to see the world scientific chart, a perspective, in which the object
objectively. There need be no mystery about this could be given an exact situation. But a system of
word: what Cézanne wished to see was the world, or perspective is no more an accurate representation of
that part of it he was contemplating, as an object, what the eye sees than a Mercator’s projection is what
without any intervention either of the tidy mind or the the world looks like from Sirius. Like the map, it
untidy emotions. His immediate predecessors, the serves to guide the intellect; perspective does not give
Impressionists, had seen the world subjectively that us any glimpse of the reality.
is to say, as it presented itself to their senses in
various lights, or from various points of view. Each One might conclude from the history of art that reality
occasion made a different and distinct impression on in this sense is a will-o’-the-wisp, an actuality we can
their senses, and for each occasion there must see but never grasp. Nature, as we say, is one thing,
necessarily be a separate work of art. But Cézanne art quite another. But Cézanne, though he was
wished to exclude this shimmering and ambiguous familiar with the “art of the museums” and respected
surface of things and penetrate to the reality that did the attempts of his predecessors to come to terms with
not change, that was present beneath the bright but nature, did not despair of succeeding where they had
deceptive picture presented by the kaleidoscope of the failed that is to say, in “realizing” his sensations in

e
senses. the presence of nature.

Great revolutionary leaders are people with a single

ri b ut Material used in this test passage has been adapted from the
following source:

t
and a simple idea, and it is the very persistency with

s
H. Read, A Concise History of Modern Painting. ©1968 by The
which they pursue this idea that endows it with

i
Herbert Read Discretionary Trust.
power. But let us ask why, in the long history of art,

wish to see the world objectively. We know, for

N t D
it had never previously happened that an artist should

o
example, that at various stages in the history of art
there have been attempts to make art “imitative”; and
59. Information in the passage suggests that the
author probably believes that the act of
interpretation:

Do
not only Greek and Roman art, but the Renaissance of A) allows the artist to view the world more
Classical art in Europe, were periods of art possessed objectively.
by a desire to represent the world “as it really is.” But B) is an almost unavoidable component of the act of
there always intervened between the visual event and seeing.
the act of realizing the vision an activity which we
can only call interpretative. This intervention seemed C) was first popularized by Cézanne.
to be made necessary by the very nature of D) became the dominant force in art many years after
perception, which does not present to the senses a flat Cézanne.
two-dimensional picture with precise boundaries but a
central focus with a periphery of vaguely
apprehended and seemingly distorted objects. The
artist might focus on a single object, say a human
figure or even a human face; but even then there were
problems such as that of representing the solidity of
the object, its place in space.

In every instance before Cézanne, in order to solve


such problems the artist brought in extra-visual

Sharing or reproducing this material in any form is a violation of the AAMC copyright Page 28 of 65

Purchased by: Yuri Brito On: 7/18/2012


Document # 6973555

60. Which of the following statements best 63. It can most reasonably be concluded from the
summarizes the central thesis of the passage? passage that Cézanne’s work exerted a powerful
influence because Cézanne:
A) For the Impressionists, each sensory occasion
required a separate work of art. A) pursued the concept of objectivity with
persistence.
B) The use of perspective prevents artists from
effectively interpreting reality. B) brought extra-visual faculties into his work.
C) Cézanne tried to solve the problem of C) expanded the concept of interpretation.
interpretation by attempting to view the world D) painted scenes as they were presented to his
objectively. senses.
D) Before Cézanne, many periods of art reflected a
desire to represent the world “as it really is.” 64. If it were discovered that Cézanne learned the
concept of objective painting from another artist,
61. The author’s suggestion that reality in art before this finding would challenge the contention that
Cézanne had been a “will-o’-the-wisp” can most Cézanne:
reasonably be interpreted to mean that artists A) pursued goals similar to those of Greek and
before Cézanne had not: Roman art.
A) attempted to imitate reality faithfully. B) was the founder of the modern movement in art.

e
B) agreed about the value of Impressionism.

t
C) respected the attempts of his predecessors to come

u
C) been able to perceive reality. to terms with nature.
D) found a way to depict reality effectively.

62. The author’s assertion that Greek, Roman, and

is t ri b D) was familiar with “the art of the museums.”

65. The author’s comparison of a system of

N
Renaissance art tried to represent the world
accurately is:

ot D
A) illustrated in the passage by examples of specific
perspective to a map is most likely intended to
show that:
A) maps are more accurate than even the most

Do
works of art. realistic artistic depictions of the world.
B) not supported by evidence in the passage. B) systems of perspective have grown more
sophisticated over the years, while mapmaking has
C) supported in the passage by a discussion of the
not.
nature of perception.
C) both afford a clearer view of the reality beneath
D) contradicted by evidence later in the passage.
everyday objects.
D) both are intellectual methods of perceiving the
world.

Sharing or reproducing this material in any form is a violation of the AAMC copyright Page 29 of 65

Purchased by: Yuri Brito On: 7/18/2012


Document # 6973555

Passage III Pennsylvania, their age, nor their size matter. Some
current we cannot explain passes through this
When I saw it first, it was a green and sleeping bud, primitive life. Each with a share of communal
raising itself toward the sun. Ants gathered aphids knowledge, all are somehow one plant.
and sap around the unopened bloom. A few days
later, it was a tender young flower with a pale green Sometimes you can hear the language of the earth in
center, a troop of silver-gray insects climbing up and water, trees, emanating from mosses, seeping through
down its stalk. Over the summer this sunflower the soil. Once, in the redwood forest, I felt something
became incredibly beautiful, subtly turning its face like a heartbeat, a hardly perceptible current that
daily, always toward the sun, its black center alive stirred a kinship and longing in me, a dream barely
with a deep blue light, as if flint had sparked an remembered. Once, on a calm beach, I heard an
elemental fire there, in community with rain, mineral, ocean storm booming from afar, revealing the
mountain air, and sand. disturbance at its center, telling about the rough water
that would arrive.
As summer changed from green to yellow, new
visitors came daily: lace-winged flies, bees with legs Tonight I watch the sky, thinking of the people who
fat with pollen, grasshoppers with clattering wings came before me and their knowledge of the placement
and desperate hunger, and other lives too small or of stars, people who watched the sun long and
hidden for me to see. This plant was a society carefully enough to witness the angle of light that
undergoing constant change, great and diverse, touched a stone just once a year. Without written

e
depending on light and moisture. records, they registered the passage of the gods of

Changes also occurred in the greater world of the


plant. One day, rounding a bend in the road, I

ri b ut night, noting fine details of the world around them


and the immensity above them. Whichever road I

t
follow, I walk in the land of many gods. Behind me,

s
encountered the disturbing sight of a dead horse, my ancestors say “Be still. Watch and listen. You
black against a hillside, eyes rolled back. Another

ot
day I was nearly lifted by a sandstorm so fierce and
hot that I had to wait for it to pass before I could
D i are the result of the love of thousands.”

Material used in this test passage has been adapted from the

N
return home. It swept away the faded dried petals of following source:
L. Hogan, Dwellings: A Spiritual History of the Living World.
the sunflower. Then the birds arrived to carry the

Do
©1995 by L. Hogan.
seeds to the future.

In one plant in one season a drama of need and 66. The author seems to be trying to understand:
survival was enacted. Hungers were filled; insects A) the beliefs of primitive peoples from the
coupled; there was escape, exhaustion, and death. An perspective of an anthropologist.
outsider, I never learned the sunflower’s golden B) the interactive balance among species from the
language. An old voice from gene or cell taught the perspective of an ecologist.
plant to oppose the pull of gravity and find its way
upward, to open. A certain knowing instinct, C) the orderly recurrence of natural forces from the
intuition, necessity directed the seed-bearing birds perspective of a poet.
to ancestral homelands they had never seen. D) the genetic regulation of behavior from the
perspective of a biologist.
There are other summons, some even more
mysterious than the survival journeys of birds and
insects. Once a century, among their canopy of sunlit
green, all bamboo plants of a certain kind flower on
the same day. Not the plants’ location, in a steamy
Malaysian jungle or a suburban garden in

Sharing or reproducing this material in any form is a violation of the AAMC copyright Page 30 of 65

Purchased by: Yuri Brito On: 7/18/2012


Document # 6973555

67. According to the author’s account, the regularity 69. Which of the following ideas about humans is
of biological cycles indicates: clearly NOT assumed in the passage?
A) a response to the competition for resources among A) Humans lack the sensory means to detect some
similar species. intraspecies messages.
B) a special mode of communication among the B) Humans have always attempted to understand
members of each species. natural occurrences.
C) the adaptation of each species to a unique niche in C) Humans are capable of existing in harmony with
its habitat. other species.
D) an inherent sensitivity to particular environmental D) Humans will eventually satisfy their curiosity
changes. about nature.

68. One can infer from the passage that for the 70. The discussion suggests that the author considers
author, the horse and the sandstorm both: the appropriate relationship of humans to other
animals to be that of:
A) present mysteries beyond human comprehension.
A) benefactor to recipient.
B) are aspects of the world that mar its beauty.
B) scientist to subject.
C) indicate the cruel indifference of nature.
C) student to teacher.

e
D) have necessary roles in a perpetual drama.

ri b ut D) parent to child.

D is t
N ot
Do

Sharing or reproducing this material in any form is a violation of the AAMC copyright Page 31 of 65

Purchased by: Yuri Brito On: 7/18/2012


Document # 6973555

Passage IV their automated production process, encouraging


workers at all stages of their operation to demonstrate
Both employers and workers are challenged by expertise and responsibility.
technological innovations, international trade,
deregulation, and changes in the nature and structure High-performance work systems are most successful
of work. Their responses to these challenges indicate when training and work reforms are bundled.
their choice of three roads to the new economy. The Similarly, workers find that their general education,
low road follows the historic path of mass production, occupational preparation, and access to training on
emphasizing downsizing, outsourcing, and low-skill the job are complementary in their effect on earnings.
employees as ways to cut labor costs. Eventually, this Workers who receive formal company training
approach, if the norm, must limit a nation's economic command higher wages than do similar workers who
competitiveness, living standard, and income equity. attend only vocational school or receive informal on-
the-job instruction. Workers who use computers on
The high road acknowledges the growing value of the job also earn more than do those of the same
investment in highly skilled employees who can react education level who do not use computers at work.
quickly to changing technologies and markets. It Moreover, the earning difference increases with the
presupposes shared power and long-term goals. Only level of technological competence.
dominant firms can afford to commit resources to
training and keeping employees by providing full For the United States to compete in an eventual global
benefits with high wages. Such firms tend to be economy based on skilled workers and quality

e
protected from domestic or international competitors products, additional employer investment in training
by technological advantages, large-scale production,
or government regulations. Currently, high-road firms
account for perhaps 20 percent of employees in the

ri b ut is needed now. Policies at all levels should encourage


the coordination of employer-provided training and
broader schooling. Such policies will realize the
United States.

D is t highest returns in terms of personal income,


adaptation to an increasingly volatile labor market,

t
About 40 percent of U.S. workers receive no formal and efficiency in the transmission of changing skill

o
training beyond a high-school education. They must requirements from workplaces to schools. Although

N
submit to the contingencies of low-road employment, for a particular job, employer-based training or
remaining at the periphery of the new economy. The vocational preparation can substitute for generalized

Do
remaining 40 percent of the workforce slog along the schooling, specific training degrades rapidly, and
muddy middle road, getting some advanced education narrow skills seldom transfer well to new job
or job-related training but unlikely to enter the requirements.
dynamic high-road labor market and attract
employers who would train them thoroughly to join But although high-wage, high-skill jobs create a
their core workers. demand for education and training, training does not
create high-wage jobs. Ultimately, a strategy of
The high road is not an easy course for employers to investment in human capital succeeds or flounders
take. Today's global customers and suppliers are according to the availability of high-wage, high-skill
linked by a web of standards that affect not only jobs. If investment in workers outpaces the number of
prices but extend to the quality and variety of good jobs, many very competent workers will face an
products, company organization, customer service employment market of many very undemanding jobs.
and its timeliness, and constant innovations.
Employers who meet these complex requirements use Material used in this test passage has been adapted from the
computer-based methods, which raise the level of following source:
skill needed by nonsupervisory personnel. For F. Fitzgerald, Cities on a Hill. ©1986 by F. Fitzgerald.
example, instead of checking the quality of the final
product, high-road firms integrate quality standards in

Sharing or reproducing this material in any form is a violation of the AAMC copyright Page 32 of 65

Purchased by: Yuri Brito On: 7/18/2012


Document # 6973555

71. The author is apparently concerned that 74. Which of the following situations is most likely
adherence to a policy referred to as "the low to constitute a muddy road, as the author uses the
road" will reduce the competitiveness of: term?
A) firms involved in international trade. A) Being trained in a skill that qualifies one for only a
particular job
B) the United States in particular.
B) Switching to unfamiliar procedures because of
C) technology-based economies.
technological changes
D) the less-developed nations.
C) Returning to college to upgrade one's professional
qualifications
72. Which of the following findings is most clearly
contrary to the reported influence of the use of D) Being chronically unemployed because of an
computers in the workplace? inadequate education

A) Office workers can follow computer-generated


schedules with less training than they need to 75. An employer reasons: "If I train my workers,
competitors who save money by not providing
devise their own schedules.
training will be able to attract my trained workers
B) Executives who correspond with customers by with higher salaries than I can pay." What
letter generate more business than those who rely possible solution for this employer would most
on E-mail alone. accord with the author's high road?
C) Workers using nonautomated production processes
are more efficient than workers on automated
assembly lines.

i b ut e A) Support regulatory policies that penalize firms for


failing to train workers.

t r
B) Train workers who agree to repay the tuition if

s
D) Mechanics who use computerized diagnostic

i
they leave within a set time.
methods earn less than mechanics who use

D
C) Concentrate on recruiting workers who have been

t
traditional methods.
trained by other firms.

N o
73. The author can best be viewed as an advocate of: D) Cut costs elsewhere to match the higher wages
paid by competitors.

Do
A) the repeal of regulations that protect dominant
76. The author asserts that to compete later,
firms.
employers should invest in training now and also
B) an increase in spending on the training of that training does not create high-wage jobs.
employees. Together, these assertions imply that:
C) an emphasis on high school vocational education. A) investment in training keeps costs low by
D) the use of computers in industrial production. providing a large pool of skilled workers.
B) in highly paid work, on-the-job training
compensates for educational deficiencies.
C) training is not effective unless it is supplemented
by a comprehensive education.
D) some highly trained workers may not benefit
financially from their training.

Sharing or reproducing this material in any form is a violation of the AAMC copyright Page 33 of 65

Purchased by: Yuri Brito On: 7/18/2012


Document # 6973555

Passage V villages, many with over ten thousand inhabitants.


Mobile-home parks for the elderly also proliferated
The residents of Sun City, Leisure World, and during this period along with other forms of age-
retirement communities across the United States live dedicated housing, from retirement hotels to luxury
on a frontier not a geographical but a chronological condominiums. The most original of these
frontier. Old age is hardly new, but for an entire innovations was the “life-care facility,” which offered
generation to reach old age with its membership small private living quarters, maid service, nursing
almost intact is new. Until relatively recently, death care, and meals, as well as nursing-home care when
had no more relation to old age than to any other necessary.
period of life in fact, it had less.
Gerontologists struggling to create a taxonomy for
A quarter of the people born in seventeenth-century these new forms of housing estimate that 5 percent of
France died during their first year, another quarter Americans over sixty-five now live in explicitly age-
died before the age of twenty, and a third quarter died segregated facilities and another unknown but
by age forty-five; only 10 percent reached sixty. significant percentage live in neighborhoods that are
From the seventeenth century to the nineteenth, the more or less age segregated. These locales are not
percentage of the French population over sixty just places in which the elderly happen to find each
remained constant at 8.8 percent. other, as they do in certain rural enclaves and inner-
city neighborhoods after everyone else has left. They
In the last hundred years, the demographics of are not only deliberate creations places to which

e
mortality have changed more than in the six previous retired persons have moved by choice but most of
centuries. In 1900, the average life expectancy for
U.S. children was 47.3 years. In 1980, it was 73.6

ri b ut them have now evolved from mere developers’ tracts


into communities with traditions of their own.

t
years. This startling increase was due mainly to

is
success in reducing infant, childhood, and maternal The construction of retirement villages initiated a
great debate among gerontologists. In the 1960s,

D
mortality. In addition also because of medical

t
advances longevity increased. In 1900, white males opinion was generally against this innovation. City

o
of sixty could expect 14.4 more years of life. In planners and journalists joined the professionals in

N
1978, they could expect to live 17.2 more years. As a attacking communities for retirees as ghettos for

Do
result of these and other changes, the number of marginalized, alienated people or as playgrounds that
Americans over sixty-five increased both absolutely trivialized the aged. But after visiting the residents of
and relative to the entire population. In 1900, 4 these villages with scientific sampling methods and
percent of the population was over sixty-five. In attitudinal charts, many gerontologists concluded that
1980, 25.5 million Americans, or 11.3 percent, were the elderly found in their segregated lives the
in this age group. advantages overtly and subliminally advertised in the
real-estate brochures.
Before World War II, there were no age-segregated
communities and there was no such concept as Material used in this test passage has been adapted from the
following source:
“retirement living.” In the early 1960s, when credit R. Barthes, The death of the author. ©1972 by Basic Books.
and housing materials were relatively cheap,
developers began to construct complete towns for the
retired. Lured by glossy advertisements depicting a
life of warm friendships and endless pleasures, many
retirees welcomed these complexes as a new
adventure. In the mid-1970s, while housing costs
doubled and trebled, the developers grew leery of
such grand schemes, but by that time there were,
according to one estimate, sixty-nine retirement

Sharing or reproducing this material in any form is a violation of the AAMC copyright Page 34 of 65

Purchased by: Yuri Brito On: 7/18/2012


Document # 6973555

77. A resident of seventeenth-century France who 80. What is the author’s probable reason for
died at age forty began having children at age prefacing the discussion of a twentieth-century
twenty. What percentage of the children would U.S. phenomenon with information about the
be expected to die before this parent? demographics of seventeenth-century France?
A) 10 percent A) To clarify the relationship between life expectancy
and the decision to change residences in old age
B) 25 percent
B) To counter the prevalent belief that demographic
C) 50 percent
changes have been unique to the U.S. culture
D) 75 percent
C) To support the argument that the current rate of
survival into old age is unprecedented
78. The assertion that death had less relation to old
age than to other periods of life most clearly D) To show the historical antecedents of the ongoing
suggests that: trend toward increased longevity

A) the increase in life expectancy has more to do with


health in early life than in late life. 81. According to the passage, an American born in
1980 could expect to live 26.3 years longer than
B) people in pre-industrial cultures felt less one born in 1900. Yet a white male of sixty
attachment to their infants than people do now. could expect to live only 2.8 years longer in 1978
C) people today are more concerned with long-term than in 1900. The difference in these

t e
planning than were earlier generations. expectancies can be explained as:

D) the elderly are less respected than in the past


because longevity is commonplace.

s t ri b u A) the cumulative effect of the growing number of


elderly.

i
B) evidence that geriatric care has improved since
1978.

D
79. New arrivals at one retirement community were

t
told by its director: “You are like pilgrims C) a result of the longer life span of women than of

o
crossing the ocean to take up a new life.” This men.

N
simile implicitly supports the author’s
D) the difference between longevity and life

Do
assumption that those moving to retirement
communities feel: expectancy.

A) ambivalence about the wisdom of breaking with


the past.
B) satisfaction at becoming independent of their
families.
C) relief at leaving situations that had become
difficult.
D) optimism about being among those with similar
goals.

Sharing or reproducing this material in any form is a violation of the AAMC copyright Page 35 of 65

Purchased by: Yuri Brito On: 7/18/2012


Document # 6973555

Passage VI thought, also had direct experience with children in an


educational setting, and both contributed profoundly
No matter how noble the effort, the burden of proof useful principles to the field of education. Yet the
always lies with the reformer. Many empirically conclusions of both about the need to consider
sound proposals to increase the effectiveness of developmental level are opposed by advocates of
elementary schools in the United States have been Generalized School Readiness.
dismissed with the response, “If it is so necessary,
why has the need not been recognized before?” To One must wonder about the experience these self-
counter this response, a reformer should make clear proclaimed experts have had with children. Their
that a problem has been identified. description of a child learning to draw, for example,
assumes a struggle from stage to stage. Most modern
If the condition addressed has not been completely observers of children think that if a task is
and clearly established as a problem, those concerned developmentally appropriate and has personal
should ensure that it is accurately measured. The meaning for a child, it is approached as a pleasing
appropriate instrument for measuring educational challenge, not a struggle.
effectiveness is a test noted for its reliability and
validity. If the researchers believe that no existing In the literature promoting their approach, the
test is adequate, they should develop their own test. advocates of generalized readiness are clearly
Since the burden of proof for their methods is then directing their appeal to school administrators.
focused on their instrument, sincere reformers will be Parents who do not understand their “readiness”

e
very serious about establishing its credentials. concept are dismissed as “uncaring.” Teachers who

When a proposed intervention is not justified in the


most minimal fashion, the public has to wonder why

ri b ut question it are described as “uninitiated,” in the sense


that someday they will accept it. Yet this literature
expresses no doubt that the administrators will
not. It is thus reasonable to be suspicious of the
promoters of the Generalized School Readiness

D is t cooperate with them in ensuring that their viewpoint


prevails. An administrator wise enough to adopt the

t
Program. What is their motivation? Are they agents readiness program is promised higher percentages on

o
of an unfriendly power bent on “dumbing down” U.S. standardized tests and more content teachers.

N
education? Are educational entrepreneurs trying
cynically to profit from the general dissatisfaction With comparative data on the results of alternative

Do
with the nation’s schools? approaches as ambiguous as they are in the U.S., the
odds favor acceptance by a school system of a poorly
Such speculations may appear to border on the researched but slickly presented program. Readiness,
absurd; however, stranger motivations have been although a confused approach, is easily implemented
discovered. It is more useful, however, to assume because its promoters are positioned to move
that the promoters, wishing to keep their business immediately. Developmentally appropriate
financially solvent, have opted not to address school- instruction, which parents are likely to judge the more
based problems from the viewpoint of children, or reasonable approach, appears to be hard to sell to
parents, or even teachers. They are merely following decision makers concerned with uniformity. In the
the usual practice at the professional level of long run, however, it is the forgotten parents and the
education of treating learning as an abstraction that children themselves who will pay for the
has little to do with the learner. This outlook is one short-sighted ambition of this policy.
that Jean Piaget, John Dewey, and A. L.
Gesell theoreticians with empirical evidence about Material used in this test passage has been adapted from the
following source:
children’s intellectual development all worked to R. Brogan, Generalized school readiness. ©1962 by
counter. Psychological Foundations of Education.

Piaget and Gesell, although from different schools of

Sharing or reproducing this material in any form is a violation of the AAMC copyright Page 36 of 65

Purchased by: Yuri Brito On: 7/18/2012


Document # 6973555

82. The author apparently considers a theory of 85. One can infer from the passage that teaching in
education that “has little to do with the learner”: the Generalized School Readiness Program is to
developmentally appropriate teaching as:
A) practical but unethical.
A) breaking farmland with a hand plow is to
B) ethical but not generally accepted.
preparing the fields by riding on a mechanized
C) generally accepted but ineffective. cultivator.
D) effective but impractical. B) sending a yearly form letter to one’s acquaintances
is to sending personal notes to one’s friends.
83. The author argues that the reason for the C) casting a line into a lake with a fishing rod is to
approach taken by promoters of the Generalized fishing with a net behind a boat.
School Readiness Program is their wish:
D) preparing a variety of dishes in a restaurant is to
A) to control the education of U.S. consumers. cooking a family meal at home.
B) to defraud the schools of a great deal of money.
C) to promote the success of a corporate sponsor. 86. The performance of high school students on an
examination noted for its reliability and validity
D) to win the support of school administrators. is used to predict their success in college. The
author’s views on such tests suggests that its
84. The most reasonable inference from passage scores could also be used to evaluate:

t e
statements is that administrators are relatively A) the usefulness of high school curricula.

u
reluctant to institute developmentally appropriate

b
B) the honesty of the grading in high schools.

i
instruction because:
A) it is favored by parents and therefore represents
the views of those with little understanding of
learning.

D is t r C) the appropriateness of the testing principles.


D) the probable income of the students in later life.

B) it is based on untested theories and therefore

N
requires extensive research to demonstrate its

ot 87. Suppose that the author reads a newspaper article


announcing that participation in the Generalized

Do
effectiveness. School Readiness Program has been shown to
C) it is individualized and therefore involves an promote the language skills of first-grade
inconvenient process of changing traditional children. What is the author’s most likely
methods. immediate reaction?

D) it is promoted in slick presentations and therefore A) To argue that the program is placing unreasonable
justifies skepticism about its cost effectiveness. demands on children of that age
B) To resolve to be less negative than before in
discussing the program
C) To doubt the accuracy of the report and the quality
of the research
D) To dismiss the findings as irrelevant to the goals
of education

Sharing or reproducing this material in any form is a violation of the AAMC copyright Page 37 of 65

Purchased by: Yuri Brito On: 7/18/2012


Document # 6973555

Passage VII with the traffic flow is a desirable thing. However, it


is the fact that people are driving on the left, or the
In a fundamental sense, every person is morally fact that frequently in social situations order is
responsible for deciding what is right and what is beneficial, that makes conformity commendable, not
wrong and acting accordingly. This responsibility is the fact that a government requires it.
often an uncomfortable burden, and you may wish to
be rid of it, to pass it on to someone else. But I will say a little about one peculiar
objection the distrust of individual conscience.
But responsibility for your actions is not transferable. Imagine someone throwing up his or her hands in
To say “I hereby promise to obey Fred, no matter horror and crying, “But you cannot let people decide
what he commands, right or wrong” is an immoral for themselves what is right and wrong. They could
attempt to forfeit your autonomy. After all, to say decide anything.” But if I surrender my decision-
that you had, in fact, made someone else responsible making responsibility to anyone, that person could
for your actions would mean that you had, somehow, come up with anything, too. Since someone has to
enslaved your will to someone else’s will, had made decide what is the right thing for me to do, it may as
yourself figuratively a puppet. Fortunately, it is not well be me. In fact, morally, it has to be.
possible. Having made a promise to obey, one is still
in fact free to obey or disobey, to keep or break that A government is made up of individuals who are
promise. fundamentally similar to me, and “to err is human”
applies to us all. Sometimes a government member

e
Now although it may seem rather bizarre to talk about may claim to have access to more information or
obeying Fred no matter what, it does follow that for
exactly the same reasons, you may not say, “I will
obey the government even if it tells me to do the

ri b ut expertise in a particular matter, in which case I will


listen to that person and may very well go along with

t
him or her because I believe that to be a strategy more

s
wrong thing.” You cannot hand over your autonomy likely to achieve a satisfactory result. Certainly, if
willy-nilly to Fred or the government or anyone else.

t
If you are told to do something, you must examine it

o D
to see if it is right or wrong before deciding whether
i you are ignorant, consult an expert, but when to
consult, whom to consult, and how to decide between
conflicting experts is still the individual’s problem.

N
to obey it or not. And so is the final decision.

Do
Therefore, if the government requires a moral person Material used in this test passage has been adapted from the
to act in a particular fashion and that person complies, following source:
P. D. Jewell, By What Authority? Anarchism, the State and the
it is because the person believes it to be the right thing Individual. ©1983 by Amanda.
to do, not simply because the government requires it.
That the government does require a particular
behavior may be an important factor to be taken into
account, but the final arbiter is the individual.

No government, no body of people, no position, no


individual can have moral authority over any other
individual.

Of course, the responsible individual will often have


to take into account what other individuals are doing
when that individual is calculating the consequences
of her or his action. The fact that the government has
decreed “Everyone shall drive on the left” may result
in people so arranging themselves, and conformity

Sharing or reproducing this material in any form is a violation of the AAMC copyright Page 38 of 65

Purchased by: Yuri Brito On: 7/18/2012


Document # 6973555

88. In response to the objection that if individuals 91. Suppose it has been demonstrated that
obeyed their consciences, they could decide government officials are ten thousand times more
anything, the author argues that the designated likely than private citizens to reach a correct
decision makers could decide anything, too. This decision on moral issues. This new information:
response is: A) supports the assertion that deciding between
A) irrelevant, because the objector does not imply conflicting experts is the problem of the
that people should be forced to disobey their individual.
consciences. B) supports the assertion that when told to do
B) irrelevant, because the author states that something, the individual must examine it to see if
responsibility is often an uncomfortable burden. it is right or wrong.
C) relevant, because the objector does not state that C) challenges the assertion that since someone has to
politicians are better decision makers than are decide the right thing for the individual to do, it
other individuals. might as well be that individual.
D) relevant, because the author implies that D) challenges the assertion that responsibility is often
politicians ought to obey their consciences, too. an uncomfortable burden that the individual may
wish to abandon.
89. Suppose that there are some people who are
psychologically incapable of breaking their 92. In countries in which the law says to drive on the

t e
promises. This new information: left, people generally find it beneficial to drive on

u
the left, and in countries in which the law says to
A) supports the author’s assertion that it is impossible

i b
drive on the right, people generally find it

r
to surrender responsibility.

t
beneficial to drive on the right. This fact:

s
B) supports the author’s assertion that in decision

i
A) supports the central thesis of the passage.
making, other people matter.
C) challenges the author’s assertion that it is

N
impossible to surrender responsibility.

ot D
D) challenges the author’s assertion that in decision
B) neither supports nor weakens the central thesis of
the passage.
C) is contrary to the central thesis of the passage.

Do
making, other people matter. D) indicates that the central thesis of the passage
requires modification.
90. Which of the following beliefs would be most in
agreement with the theme of the passage?
A) Laws have no legitimacy except insofar as they
accord with the will of the governed.
B) Laws are necessary to prevent people from
deciding moral issues for themselves.
C) Laws should be obeyed because obedience
promotes order and so is generally beneficial.
D) Lawmakers should consult experts rather than rely
on their own fallible judgment.

Sharing or reproducing this material in any form is a violation of the AAMC copyright Page 39 of 65

Purchased by: Yuri Brito On: 7/18/2012


Document # 6973555

Writing Sample
Time: 60 minutes
2 Prompts, separately timed:
30 minutes each

This is a test of your writing skills. The test consists of two parts. You will have 30 minutes to complete each
part. Use your time efficiently. Before you begin writing each of your responses, read the assignment carefully
to understand exactly what you are being asked to do. Because this is a test of your writing skills, your
response to each part should be an essay of complete sentences and paragraphs, as well organized and clearly
written as you can make it in the time allotted.

i b ut e
D is t r
N ot
Do
This document has been encoded to link this download to your member account. The AAMC and its Section
for the MCAT hold the copyrights to the content of this Practice Test. Therefore, there can be no sharing or
reproduction of materials from the Practice Test in any form (electronic, voice, or other means). If there are any
questions about the use of the material in the Practice Test, please contact the MCAT Information Line (202-
828-0690).

Purchased by: Yuri Brito On: 7/18/2012


Document # 6973555

93. Consider this statement:

Businesses succeed by taking advantage of consumers’ weaknesses.

Write a unified essay in which you perform the following tasks. Explain what you think the above
statement means. Describe a specific situation in which businesses succeed without taking advantage of the
consumers’ weaknesses. Discuss what you think determines whether or not businesses take advantage of
consumers’ weaknesses in order to succeed.

i b ut e
D is t r
N ot
Do

Sharing or reproducing this material in any form is a violation of the AAMC copyright Page 41 of 65

Purchased by: Yuri Brito On: 7/18/2012


Document # 6973555

94. Consider this statement:

Education makes everyone equal.

Write a unified essay in which you perform the following tasks. Explain what you think the above
statement means. Describe a specific situation in which education does not make everyone equal. Discuss
what you think determines whether or not education makes everyone equal.

i b ut e
D is t r
N ot
Do

Sharing or reproducing this material in any form is a violation of the AAMC copyright Page 42 of 65

Purchased by: Yuri Brito On: 7/18/2012


Document # 6973555

Biological Sciences
Time: 70 minutes
Questions: 95 - 146

Most questions in the Biological Sciences test are organized into groups, each containing a descriptive passage.
After studying the passage, select the one best answer to each question in the group. Some questions are not
based on a descriptive passage and are also independent of each other. If you are not certain of an answer,
eliminate the alternatives that you know to be incorrect and then select an answer from the remaining
alternatives. Indicate your selected answer by marking the corresponding answer on your answer sheet. A
periodic table is provided for your use. You may consult it whenever you wish.

i b ut e
D is t r
N ot
Do
This document has been encoded to link this download to your member account. The AAMC
and its Section for the MCAT hold the copyrights to the content of this Practice Test. Therefore,
there can be no sharing or reproduction of materials from the Practice Test in any form
(electronic, voice, or other means). If there are any questions about the use of the material in the
Practice Test, please contact the MCAT Information Line (202-828-0690).

Purchased by: Yuri Brito On: 7/18/2012


Document # 6973555

1
Periodic Table of the Elements 2
H He
1.0 4.0
3 4 5 6 7 8 9 10
Li Be B C N O F Ne
6.9 9.0 10.8 12.0 14.0 16.0 19.0 20.2
11 12 13 14 15 16 17 18
Na Mg Al Si P S Cl Ar
23.0 24.3 27.0 28.1 31.0 32.1 35.5 39.9
19 20 21 22 23 24 25 26 27 28 29 30 31 32 33 34 35 36
K Ca Sc Ti V Cr Mn Fe Co Ni Cu Zn Ga Ge As Se Br Kr
39.1 40.1 45.0 47.9 50.9 52.0 54.9 55.8 58.9 58.7 63.5 65.4 69.7 72.6 74.9 79.0 79.9 83.8
37 38 39 40 41 42 43 44 45 46 47 48 49 50 51 52 53 54
Rb Sr Y Zr Nb Mo Tc Ru Rh Pd Ag Cd In Sn Sb Te I Xe
85.5 87.6 88.9 91.2 92.9 95.9 (98) 101.1 102.9 106.4 107.9 112.4 114.8 118.7 121.8 127.6 126.9 131.3
55 56 57 72 73 74 75 76 77 78 79 80 81 82 83 84 85 86
Cs Ba La* Hf Ta W Re Os Ir Pt Au Hg Tl Pb Bi Po At Rn
132.9 137.3 138.9 178.5 180.9 183.9 186.2 190.2 192.2 195.1 197.0 200.6 204.4 207.2 209.0 (209) (210) (222)
87 88 89 104 105 106 107 108 109 110 111 112 114 116
Fr Ra Ac† Rf Db Sg Bh Hs Mt Ds Uuu Uub Uuq Uuh
(223) (226) (227) (261) (262) (266) (264) (277) (268) (281) (272) (285) (289) (289)

58 59 60 61 62 63 64 65 66 67 68 69 70 71

t e
* Ce Pr Nd Pm Sm Eu Gd Tb Dy Ho Er Tm Yb Lu

u
140.1 140.9 144.2 (145) 150.4 152.0 157.3 158.9 162.5 164.9 167.3 168.9 173.0 175.0

b
90 91 92 93 94 95 96 97 98 99 100 101 102 103

i
† Th Pa U Np Pu Am Cm Bk Cf Es

r
Fm Md No Lr

t
232.0 (231) 238.0 (237) (244) (243) (247) (247) (251) (252) (257) (258) (259) (260)

t D is
N o
Do

Purchased by: Yuri Brito On: 7/18/2012


Document # 6973555

Passage I

The axanes are a family of compounds that contain a


perhydroindan ring system; Figure 1 shows the
structures of two axanes, compounds 1 and 2.

Figure 1 Axanes

A multistep synthesis leading to the axanes begins


with the conversion of Compound 3 into Compound
8, which can be further functionalized to the target
axanes. Figure 2 shows the synthesis of Compound 8.

i b ut e
D is t r
t
Figure 2 Synthesis of Compound 8 (Et = ethyl;

o
Ph = phenyl)

Do N Step 1 involves a 1,4-addition (Michael addition) of


3-butenylmagnesium bromide to Compound 3,
yielding an adduct that reforms the endocyclic double
bond with the loss of ethoxide during the acidic
workup. Step 2 involves the treatment of Compound
4 with lithium dimethylcuprate to form a Michael
adduct that hydrolyzes in acid to Compund 5. The
ozonolysis of Compound 5 in Step 3 and a reductive
workup produces Compound 6, which undergoes a
Wittig reaction with Ph3P=CHCO2Et in Step 4 to
yield Compound 7. Compound 7 undergoes a base-
catalyzed intramolecular cyclization in Step 5 to yield
Compound 8 in 30% overall yield from Compound 3.

Sharing or reproducing this material in any form is a violation of the AAMC copyright Page 45 of 65

Purchased by: Yuri Brito On: 7/18/2012


Document # 6973555

95. According to the Cahn-Ingold-Prelog priority 98. The base in Step 5 of the synthesis abstracts a
rules, which group bonded to the chiral carbon proton from which of the labeled (a-d) carbon
atom (x) in Figure 1 has the highest priority? atoms of Compound 7 (Figure 2)?
A) The H atom A) a
B) The ring system B) b
C) The NHCHO group C) c
D) The isopropyl group D) d

96. What reagent yields the double bond in 99. On the basis of the bonding and functionality
Compound 1 when it reacts with an analog of present in Compound 7, a student can classify the
Compound 8? compound as:
A) Ph3P=CH2 A) a saturated keto ether.
B) O=CH2 B) a saturated keto ester.
C) (CH3)2CuLi C) an unsaturated keto ether.
D) CH3MgBr D) an unsaturated keto ester.

t e
97. What is the maximum possible number of

u
stereoisomers of Compound 1?
A) 4
B) 8

is t ri b
D
C) 16
D) 32

N ot
Do

Sharing or reproducing this material in any form is a violation of the AAMC copyright Page 46 of 65

Purchased by: Yuri Brito On: 7/18/2012


Document # 6973555

102. The concentration of the protein cyclin rises and


These questions are not based on a descriptive falls during the cell cycle as shown in Figure 1.
passage and are independent of each other.

100. A student uses thin layer chromatography on


silica plates to monitor the progress of the
reaction below. Does the product have a higher
or lower Rf than the starting material?

Figure 1 Changes in the concentration of


cyclin during phases of the cell cycle

A) Higher, because the product is more polar than the What mechanism could account for this oscillation
starting material of cyclin protein concentration?

B) Higher, because the product is less polar than the A) Replication of the cyclin gene during S phase of
starting material interphase

C) Lower, because the product is less polar than the B) Segregation of chromosomes carrying the cyclin
starting material genes during mitosis

D) Lower, because the product is more polar than the


starting material

i b ut e C) Translation of cyclin mRNA in interphase and


proteolysis of cyclin protein in mitosis

t r
D) Translation of cyclin mRNA in mitosis and

is
101. Which of the following alkyl halides is most proteolysis of cyclin protein in interphase

D
readily prepared by a reaction between the

t
corresponding alcohol and concentrated 103. Embryonic mouse cells divide every 10 hours at

N
hydrochloric acid?
A) Isopropyl chloride
o 37oC. How many cells would be produced from
an egg after three days?

Do
B) Methyl chloride A) Fewer than 50

C) sec-Butyl chloride B) Between 50 and 500

D) tert-Butyl chloride C) Between 500 and 5000


D) More than 5000

Sharing or reproducing this material in any form is a violation of the AAMC copyright Page 47 of 65

Purchased by: Yuri Brito On: 7/18/2012


Document # 6973555

Passage II

Taxol (1) is a naturally occurring compound that has been adopted as a drug for the treatment of a variety of
cancers. A need exists for an efficient synthesis of taxol because it is found only in small quantities in a rare
tree.

Taxol (1)

t e
Figure 1 shows a synthetic approach to ring A. Compound 2 undergoes a Grignard reaction with CH3MgBr,

u
followed by dehydration to produce Compound 3. The carboethyoxy group in Compound 3 is converted into

i b
the primary alcohol 4 by diisobutylaluminum hydride. Compound 4 is converted into its acetate 5, which

Taxol.

D i t r
undergoes a regiospecific Diels–Alder reaction with 6 to produce 7. The reaction of Compound 7 with

s
alcoholic KOH yields Compound 8, which can serve as a key intermediate for the formation of the B ring in

N ot
Do

Sharing or reproducing this material in any form is a violation of the AAMC copyright Page 48 of 65

Purchased by: Yuri Brito On: 7/18/2012


Document # 6973555

i b ut e
D is t r
N ot
Do Figure 1 Synthetic scheme

Sharing or reproducing this material in any form is a violation of the AAMC copyright Page 49 of 65

Purchased by: Yuri Brito On: 7/18/2012


Document # 6973555

107. Compound 7 is a mixture of two isomers that


104. What are the configurations at C-5 and C-7, are shown by which pair of structures below?
respectively, in Taxol? A)
A) R and R
B) S and S
C) S and R
D) R and S
B)
105. Which of the following compounds is formed
by the Grignard reaction shown in Figure 1?
A)

C)

B)

i b ut e
r
D)

C)

D is t
N ot
Do
108. The IR spectrum of Compound 8 shows
D) hydroxyl and carbonyl absorptions, respectively,
at approximately:
A) 1700 and 2250 cm–1.
B) 1700 and 3400 cm–1.
C) 3400 and 2250 cm–1.
106. The preparation of the Grignard reagent
involves stirring which of the following reagents D) 3400 and 1700 cm–1.
in diethyl ether (Et2O)?
A) CH3Br and Mg
B) CH4 and MgBr2
C) CH3Mg and Br2
D) CH2Br2, Mg, and Br2

Sharing or reproducing this material in any form is a violation of the AAMC copyright Page 50 of 65

Purchased by: Yuri Brito On: 7/18/2012


Document # 6973555

Passage III 109. What process would be most disrupted by an


inflammation of the colon?
Crohn’s disease and ulcerative colitis are two forms
of inflammatory bowel disease, which differ in A) Digestion
several respects. Crohn’s disease may occur in any B) Absorption of nutrients
part of the gastrointestinal tract while ulcerative
C) Absorption of water
colitis is confined to the colon (large intestine).
Crohn’s disease may involve all layers of the tract D) Secretion of digestive enzymes
while ulcerative colitis affects only the mucosa, the
inner lining of the colon. Abdominal pain and 110. Normally the immune system avoids attacking
diarrhea following a meal are signs of Crohn’s the tissues of its own body because:
disease, but a progressive loosening of a bloody stool
is the first symptom of ulcerative colitis. A) a special intracellular process recognizes only
foreign antigens.
Management of inflammatory bowel disease is B) the body does not make any antigens that the
achieved by drug therapy to suppress the immune system could recognize.
inflammation which leads to diarrhea, but there is no
C) it changes its antibodies to be specific only to
known cure. The cause of inflammatory bowel
foreign antigens.
disease is controversial. Genetic, pathogenic, and
immunogenic theories have all been advanced. D) it suppresses cells specific to the body's own

e
antigens.
Inflammatory bowel disease tends to run in families,
with 20% of patients having a relative with the
disorder. But if inflammatory bowel disease is

ri b ut 111. An ulcer that penetrated the wall of the intestine

t
would allow the contents of the gastrointestinal

s
genetic, it is not inherited in a simple Mendelian way.

i
tract to enter:

D
A) the perineum.

t
Some research suggests that inflammatory bowel

o
disease is an autoimmune disease. An antigen in the B) the peritoneal cavity.

N
body, perhaps in the digestive tract, is recognized as
foreign by the immune system. This antigen may C) the pleural cavity.

Do
then stimulate the body’s defenses to produce an D) the lumen of the intestine.
inflammatory response that continues without control.
112. If the genetic and autoimmune theories of
There is also a lingering suspicion that the inflammatory bowel disease are true, then the
inflammation is triggered by some bacterium or other gastrointestinal antigen being targeted by the
organism that takes up residence in the immune system is probably on:
gastrointestinal system. This theory has had a
resurgence since the discovery that the bacterium A) the chromosomes carrying the genes for the
Helicobacter pylori may play a causal role in gastric disease.
ulcers. B) part of the DNA segments constituting the genes
for the disease.
C) stretches of the mRNA's coded for by the genes
for the disease.
D) the surface of the proteins encoded by the genes
for the disease.

Sharing or reproducing this material in any form is a violation of the AAMC copyright Page 51 of 65

Purchased by: Yuri Brito On: 7/18/2012


Document # 6973555

113. The fact that there appears to be a genetic


component to inflammatory bowel disease, but
that it does not show clear Mendelian
inheritance ratios suggests any of the following,
EXCEPT:
A) the gene for the disease has incomplete
penetrance.
B) the gene for the disease has limited expressivity.
C) the disease is polygenic.
D) the gene for the disease is recessive.

i b ut e
D is t r
N ot
Do

Sharing or reproducing this material in any form is a violation of the AAMC copyright Page 52 of 65

Purchased by: Yuri Brito On: 7/18/2012


Document # 6973555

Passage IV
Vitamin E, an effective anti-oxidant, was given to 2
Fats are known to affect blood flow. Research was groups to reduce in vivo oxidation of the ingested
conducted to examine the effects of a 20-carbon fatty acids. Both the fatty acid and the placebo were
polyunsaturated fatty acid on blood flow through the given at 2 g/10 kg body weight/day; vitamin E was
skin. Skin was chosen because blood flow could be given at 100 mg/10 kg body weight/day. The
measured easily and without discomfort to the supplements were packaged in identical gelatin
subjects. capsules and taken before meals. Subjects were
unaware of which supplement they were ingesting.
Male college students ranging from 18-28 years of
age were randomly divided into 4 treatment groups of Blood flow through the skin of each subject’s arm
10 subjects each. Prior to participation in the study, was measured twice; immediately before and
subjects were screened for health conditions, immediately after 60 days of dietary
medications, and/or dietary practices that would bias supplementation. Each blood-flow measurement was
the collected data. Subjects were instructed not to taken at the same time of day and at a skin
alter their habits or lifestyles during the experiment. temperature of 32oC. Subjects were required to fast
for 12 hours and rest quietly in the laboratory for 30
Each treatment group was given 1 of 4 dietary minutes before skin blood flow was measured. The
supplements (Table 1). results are shown in Figure 1.

e
Table 1 Dietary Supplements

placebo

ri b ut
t
placebo + vitamin E

is
fatty acid

D
fatty acid + vitamin E

N ot
Do
Figure 1 Effect of dietary supplements on skin blood
flow

Sharing or reproducing this material in any form is a violation of the AAMC copyright Page 53 of 65

Purchased by: Yuri Brito On: 7/18/2012


Document # 6973555

114. The most likely explanation for the difference in 116. An alternative method for examining the effects
skin blood flow between the fatty acid group of fatty acids on blood flow would be to
and the fatty acid + vitamin E group in Figure 1 measure changes in blood pressure. If blood
is that: pressure were measured, one would predict that
it would be lowest in which of the following?
A) vitamin E alone reduces skin blood flow more
than fatty acids alone. A) Heart
B) vitamin E alone increases skin blood flow more B) Arteries
than fatty acids alone. C) Arterioles
C) the products of fatty acid oxidation reduce skin D) Capillaries
blood flow.
D) unoxidized fatty acids reduce skin blood flow. 117. To interpret the results, the researchers must
assume that:
115. It was hypothesized that the decrease in blood A) fatty acids have no effect on skin blood flow.
flow to the skin resulted from a change in the
activity of the sympathetic nerves to the skin. B) vitamin E reduces skin blood flow.
Which of the following observations would C) the subjects did not alter their habits during the
support this hypothesis? study.
A) A change in the norepinephrine content of blood

e
D) blood pressure differed between the 2

t
draining from the skin measurements.
B) In vitro contraction of the smooth muscle in skin
blood vessels in response to acetylcholine

s t ri b u 118. In the design of the experiment, all of the

i
C) A lack of epinephrine receptors in skin blood following factors were controlled EXCEPT:

D
vessels A) skin temperature.
D) In vivo dilation of the skin blood vessels

N ot B) age of the subjects.


C) diurnal rhythms in physiological responses.

Do
D) skin blood flow.

Sharing or reproducing this material in any form is a violation of the AAMC copyright Page 54 of 65

Purchased by: Yuri Brito On: 7/18/2012


Document # 6973555

Passage V The 4-carbon molecules were hypothesized to arise


from the following steps:
Organic carbocations may be generated from alcohols The ethylcarbocation loses a proton to form
in the presence of strong acids or from alkenes by the ethene.
addition of a proton. Carbocations are stabilized Another CH3CH2+ carbocation reacts with the
through the inductive effect and through resonance. ethene to form a 4-carbon carbocation.
Alkyl groups bonded to the cation center also may Rearrangements contribute to the formation of
add stability through the partial overlap of filled the final products.
orbitals with an empty orbital, a concept often
referred to as hyperconjugation. Such alkyl groups 119. Which of the following concepts may be used to
have one filled orbital aligned with the empty orbital explain why the stability of carbocations
of the cation. The overlap of these orbitals allows the increases as the number of alkyl groups attached
pair of electrons in the filled orbital to reduce the to the carbocation increases?
electron deficiency of the cation center.
A) Resonance
In an early research study designed to examine the B) Steric interference
properties of carbocations, an ethyl carbocation was
C) Hyperconjugation
produced from the reaction of ethyl fluoride with
SbF5, as shown in Reaction I (unbalanced). D) Ion pairing

t e
120. Which of the following compounds is the major

u
compound isolated from Reaction 1?
Reaction I

The products of Reaction I were quenched with water,

is t ri b A)

D
then separated by gas chromatography, and identified B)
by 1H NMR. The resulting mixture of products

N
consisted primarily of molecules with four carbon
atoms. The proton NMR spectrum of the major
ot
Do
product is shown below in Figure 1.
C)

D)

Figure 1

The next most abundant product was an optically


inactive sample, later determined to be sec-butyl
alcohol.

Sharing or reproducing this material in any form is a violation of the AAMC copyright Page 55 of 65

Purchased by: Yuri Brito On: 7/18/2012


Document # 6973555

121. The ethyl carbocation may be formed when 123. All the products that were isolated from the
ethanol is heated with sulfuric acid. The first quenching of Reaction 1 should contain an
step of this reaction is: absorption in the infrared spectrum near:
A) the elimination of water from ethyl alcohol. A) 1600 cm-1.
B) the protonation of the hydroxyl oxygen of ethyl B) 1700 cm-1.
alcohol. C) 2250 cm-1.
C) the loss of a proton by ethyl alcohol. D) 3500 cm-1.
D) the loss of the hydroxyl group of ethyl alcohol.

122. The gas chromatograph trace from the workup


of Reaction 1 is shown below with the
integrated areas indicated.

What percentage of sec-butyl alcohol was

t e
isolated?
A) 1.5%
B) 3%

s t ri b u
i
C) 5%
D) 6%

N ot D
Do

Sharing or reproducing this material in any form is a violation of the AAMC copyright Page 56 of 65

Purchased by: Yuri Brito On: 7/18/2012


Document # 6973555

These questions are not based on a descriptive 126. If 1 mol of a pure triglyceride is hydrolyzed to
passage and are independent of each other. give 2 mol of RCOOH, 1 mol of R'COOH, and
1 mol of glycerol, which of the following
124. An organism that causes a human disease is compounds might be the triglyceride?
isolated and studied. Researchers conclude that A)
the organism is a bacterium rather than a virus
because the organism:
A) undergoes mutation.
B) lacks a nuclear membrane.
C) contains protein in its outermost covering.
B)
D) reproduces in a culture medium lacking host
tissue.

125. Which of the following organelles most


resembles the Golgi apparatus when an intact
eukaryotic cell is viewed under the electron
microscope? C)

A) Nucleolus
B) Mitochondrion
C) Plasma membrane

i b ut e
D) Smooth endoplasmic reticulum

D is t r D)

N ot
Do 127. The lipases catalyze the hydrolysis of fats and
other carboxylic acid esters. The lipases
illustrate the fact that:
A) some enzymes are molecules other than proteins.
B) most enzymes interact with only one specific
substrate molecule.
C) some enzymes interact with several different
substrate molecules that have similar chemical
linkages.
D) some enzymes interact with many biologically
active substrate molecules of dissimilar structures
and linkages.

Sharing or reproducing this material in any form is a violation of the AAMC copyright Page 57 of 65

Purchased by: Yuri Brito On: 7/18/2012


Document # 6973555

128. In horses, the genes for red coat color and for
white coat color are codominant. Heterozygous
horses have roan-colored coats. Consider a
roan-colored colt that has a white mother. What
could be said about the coat color of the colt’s
father?
A) It must be red.
B) It must be roan.
C) It could be either red or roan.
D) It could be either red or white.

i b ut e
D is t r
N ot
Do

Sharing or reproducing this material in any form is a violation of the AAMC copyright Page 58 of 65

Purchased by: Yuri Brito On: 7/18/2012


Document # 6973555

Passage VI
130. Which of the following statements explains
Until recently, conventional medical wisdom most plausibly why host antibodies are
attributed stomach ulcers to an excess of acid, and the ineffective against H. pylori?
treatment for ulcers consisted primarily of antacids
and dietary modification. However, a pathogenic A) Antibody proteins may be denatured in the harsh
bacterium, Helicobacter pylori, is now implicated in environment of the stomach.
most cases of stomach ulcers. Current treatment B) Antibodies are not generally effective against
employs antibiotics directed against these bacteria bacteria.
and often is successful in eradicating persistent
C) H. pylori infection may suppress the activity of the
infections.
immune system.
Several important questions about H. pylori remain D) Antibodies are not secreted from host tissues into
unanswered. It is unclear why this bacterium causes extracellular spaces.
chronic infections in some individuals but not in
others; many infected persons do not develop ulcers. 131. One difference between different strains of H.
The mode of transmission is also unknown, although pylori is that they:
people in developing countries are more frequently
infected with H. pylori than are people in developed A) attack different hosts.
countries with good sanitation. B) express different genes.

There is a relationship between H. pylori infection


and cancer. Infected individuals have a two-fold

i b ut e C) exhibit different degrees of resistance to


antibiotics.

r
increased risk of gastric cancer, although >75% of D) exist in either developed or developing countries.
patients with active infections do not develop cancer.

D i
Genetic studies of H. pylori have identified genes that

s t 132. According to the passage, the cagA gene

t
are expressed in different strains of this bacterium. product will cause:

o
One gene, vacA, encodes a toxin. Expression of
A) the disruption of host cell enzymatic activity.

N
another gene, cagA, leads to inflammation and may
be related to the genesis of gastric cancer. Although

Do
B) the disruption of host cell protein synthesis.
many individuals develop antibodies against H. pylori
antigens, these antibodies rarely eradicate the C) the movement of leukocytes into mucosal tissue.
infection; evidently, this pathogen has developed D) the vasoconstriction of arterioles in the mucosal
effective ways to elude host defenses. layer.

129. H. pylori infection may cause increased 133. Most people infected with H. pylori do not
proliferation of mucosal cells in the stomach. develop gastric cancer because they:
This may lead to gastric cancer if:
A) do not incorporate bacterial genes in their
A) genetic mutations occur in proliferating germ chromosomes.
cells.
B) have robust immune systems that defeat early
B) genetic mutations occur in proliferating somatic cancers.
cells.
C) eradicate the infection before any tumors develop.
C) the immune system fails to recognize bacterial
antigens. D) tolerate the infection without developing tumors.

D) crowded mucosal cells are likely to remain in


interphase.

Sharing or reproducing this material in any form is a violation of the AAMC copyright Page 59 of 65

Purchased by: Yuri Brito On: 7/18/2012


Document # 6973555

134. To be most effective, a gene therapy for gastric


cancer should be directed against: 135. Enzymatic activity in the stomach initiates the
A) the stomach epithelial cells that give rise to digestion of:
tumors. A) lipids.
B) the antibody producing cells of the immune B) ethanol.
system.
C) polysaccharides.
C) all the cells in the host’s body.
D) proteins.
D) all known genes in the H. pylori genome.

i b ut e
D is t r
N ot
Do

Sharing or reproducing this material in any form is a violation of the AAMC copyright Page 60 of 65

Purchased by: Yuri Brito On: 7/18/2012


Document # 6973555

Passage VII

Nematode development has been used as a model system to investigate the differentiation of cells from their
neighbors (see Figure 1). Three experiments are described here.

Figure 1 Early development, showing one-, two-, and four-cell stages (left), and segregation of fate (right) in
the nematode. The blastomeres and tissues formed by the founder cells are shown.

i b ut e
D is t r
N ot
Do
Figure 2 Results of Experiment 3. At right center are shown the cell recombinations, and the time lines at far
right indicate when the cells were separated and recombined, as well as the results.

Sharing or reproducing this material in any form is a violation of the AAMC copyright Page 61 of 65

Purchased by: Yuri Brito On: 7/18/2012


Document # 6973555

Experiment 1

To investigate the role of cell-to-cell communication, researchers separated the cells of a two-cell embryo and
cultured them independently. The cultured AB cells produced neurons and skin, but no muscle, whereas the
cultured P1 cells gave rise to all of the tissues produced by P1 cells of an intact embryo.

Experiment 2

Two-cell embryos were incubated in the presence of either cycloheximide, an inhibitor of translation, or
actinomycin D, an inhibitor of transcription. The AB cells were then isolated, washed to remove the inhibitors,
and grown in culture. AB cells from embryos treated with cycloheximide produced only neurons and skin,
whereas those from embryos treated with actinomycin D produced neurons, skin, and muscle.

Experiment 3

To investigate specification of the gut, which was thought to result from the segregation of cytoplasmic contents
during early cell divisions, cells were isolated at various intervals (as indicated on the time lines of Figure 2)
during the 15-minute four-cell stage, cultured individually or recombined in pairs, and allowed to develop. If
gut differentiation occurred, proteases were released into the culture medium. The results are shown in Figure 2
.

i b ut e
D is t r
N ot
Do

Sharing or reproducing this material in any form is a violation of the AAMC copyright Page 62 of 65

Purchased by: Yuri Brito On: 7/18/2012


Document # 6973555

139. The results of Experiment 3 indicate that gut


136. Which result of Experiment 1 supports the specification during the four-cell stage requires
hypothesis that cell-to-cell communication is cell-to-cell communication between:
involved in the determination of cell fate? A) P2 and EMS.
A) The fate of an isolated AB cell differs from that of B) AB1 and EMS.
an AB cell in an intact embryo.
C) P1 and AB1.
B) The fate of an isolated P1 cell is indistinguishable
from that of a P1 cell in an intact embryo. D) AB1 and AB2.
C) At the two-cell stage, isolated blastomeres can
divide and differentiate. 140. If the zygote contains unique cell contents that
are necessary for gut differentiation, segregation
D) Several different blastomeres can produce both of these substances during cell divisions would
neurons and muscle tissue. occur in the sequence of zygote to P1 to:
A) AB.
137. The results of Experiment 1 indicate that the
direction of signaling between the blastomeres B) EMS to E.
of a two-cell embryo is: C) P2 to EMS to E.
A) AB → P1. D) both P2 and EMS.

e
B) P1 → AB.
C) P1 → P2.

ri b ut 141. The only somatic or visceral cell-type tissue that


derives from a single blastomere is:

t
D) zygote → AB. A) neuronal.

138. The results of Experiment 2 indicate that the

t D is B) muscle.
C) gut.

o
signaling interaction at the two-cell stage

N
probably most involves which class of D) germ cell.
macromolecules?

Do
A) DNA 142. These experiments indicate that nematode cells
adopt fates different from those of their
B) Messenger RNA
neighbors during development by:
C) Ribosomal RNA
A) mechanisms that do not involve transcription.
D) Protein
B) cell separation followed by independent
development of the blastomeres.
C) both cell-to-cell signaling and segregation of the
cytoplasmic contents during division.
D) the separation of their three primary tissue layers.

Sharing or reproducing this material in any form is a violation of the AAMC copyright Page 63 of 65

Purchased by: Yuri Brito On: 7/18/2012


Document # 6973555

These questions are not based on a descriptive 146. In an experiment on the phases of the cell cycle,
passage and are independent of each other. cultures of actively dividing, cells were exposed to
radioactively labeled 2-deoxythymidine for 30
143. The following pedigree shows the occurrence of minutes, then rinsed to remove the unabsorbed label.
a very rare disease that is expressed in fewer than At various times thereafter, groups of cells were
1 in 100,000 individuals. removed from the cultures and the nuclei examined to
determine their content of radioactive material.
Results are shown in the figure below.

Which of the following is the most likely pattern


of inheritance for this disease in these
individuals?
A) The mother carried a sex-linked dominant allele for

e
the disease.
B) The mother carried a sex-linked recessive allele for
the disease.

ri b ut Based on the figure, the approximate length of what

the disease.

D is
C) Each father carried an autosomal recessive allele for

t process is indicated by the 3-13 hour interval after


treatment with radioactive 2-deoxythymidine?

t
D) Each father carried a sex-linked recessive allele for A) Mitosis

N
the disease.

o B) Meiosis

Do
C) DNA synthesis
144. The liver is different from many other organs in
that it can at least partially regenerate following D) RNA synthesis
illness or damage. This regeneration is
accomplished primarily through:
A) fission.
B) meiosis.
C) mitosis.
D) cell growth.

145. In humans, cholesterol is a precursor to:


A) insulin.
B) glycogen.
C) testosterone.
D) DNA.

Sharing or reproducing this material in any form is a violation of the AAMC copyright Page 64 of 65

Purchased by: Yuri Brito On: 7/18/2012


Document # 6973555

i b ut e
D is t r
N ot
Do

Sharing or reproducing this material in any form is a violation of the AAMC copyright Page 65 of 65

Purchased by: Yuri Brito On: 7/18/2012

Você também pode gostar